SlideShare ist ein Scribd-Unternehmen logo
1 von 36
Ph¬ng tr×nh , BÊt ph¬ng tr×nh v« tØ
Bµi 1: Gi¶i ph¬ng tr×nh
a) + = −3 3
1 2 2 1x x
+ = −
= − ⇔ + =
3 3
33
1 2 2 1
2 1 1 2
x x
y x y x
- Ph¬ng tr×nh ®îc chuyÓn thµnh hÖ

 =  = =
 
+ = + = + =  − +   ⇔ ⇔ ⇔ = =   + = − = − − + + + =      − − + = = =
33 3
3 3 3 2 2
3
1
1 21 2 1 2 1 5
21 2 2( ) 2 0( )
1 51 2
2
x y x y
x yx y x y
x y
y x x y x y x xy y vn
x y x y
- VËy ph¬ng tr×nh ®· cho cã 3 nghiÖm.
b) + − = + −2 2
1 1 (1 2 1 )x x x
§S:x=1/2; x=1
c) − + − = − + − +2
( 3 2 1) 4 9 2 3 5 2x x x x x
§S: x=2.
d)
+
− + + − = −
−
1
( 3)( 1) 4( 3) 3
3
x
x x x
x
§S: = − = −1 13; 1 5x x
e) − + − = − +2
2
1 1
2 2 4 ( )x x
x x
- Sö dông B§T Bunhia.
f) + − − = −4 1 1 2x x x
§S: x=0
Bµi 2: Gi¶i BPT:
a) + − − ≤5 1 4 1 3x x x
§S: x≥1/4
b)
− −
+ − >
− −
2
2( 16) 7
3
3 3
x x
x
x x
§K
 − ≥
⇔ ≥
− >
2
16 0
4
3 0
x
x
x
- BiÕn ®«Ø bÊt ph¬ng tr×nh vÒ d¹ng
− + − > − ⇔ − > −
− <
>
− ≥⇔ ⇔ ⇔ > − 
− < ≤  − > −
2 2
2 2
2( 16) 3 7 2( 16) 10 2
10 2 0
5
10 2 0 10 34.
10 34 5
2( 16) (10 2 )
x x x x x
x
x
x x
x
x x
- KÕt hîp §K ta cã nghiÖm cña BPT lµ > −10 34x .
c) + − > −( 1)(4 ) 2x x x .
1
d)
− −
<
2
1 1 4
3
x
x
.
§K:

− ≤ < − ≥
⇔ 
≠  < ≤

2
1
0
1 4 0 2
10
0
2
x
x
x
x
- Thùc hiÖn phÐp nh©n liªn hîp ta thu ®îc BPT
< + − ⇔ − > −

< − < 
  ≤− ≥ ⇔ ⇔ ⇔ ≤ − ≥  ≥ − > −  
 − > −
2 2
2
2 2
2 2
4 3(1 1 4 ) 3 1 4 4 3
3
44 3 0
1
1 4 0 1
2
24 3 0
3
9(1 4 ) (4 3) 4
9(1 4 ) (4 3)
x x x x
x
x
xx
x
x
xx x
x x
.
- KÕt hîp §K thu ®îc nghiÖm

− ≤ <

 < ≤

1
0
2
1
0
2
x
x
C¸ch 2:
- XÐt 2 TH:
+ Víi − ≤ < ⇔ − < −21
0. 1 4 1 3
2
x BPT x x
+ Víi < ≤ ⇔ − > −21
0 . 1 4 1 3
2
x BPT x x
e) 2 2
5 10 1 7 2x x x x+ + ≥ − −
§K:
2
5 2 5
5
5 10 1 0
5 2 5
5
x
x x
x
 − −
≤
+ + ≥ ⇔
 − +
≥

- Víi §k ®ã 2 2
5 5 10 1 36 5 10 1x x x x− + + ≤ − + + +
- §Æt 2
5 10 1; 0t x x t= + + ≥ .
- §S: x≤-3 hoÆc x≥1.
Bµi 3: T×m m ®Ó ph¬ng tr×nh sau cã nghiÖm:
2 2
1 1x x x x m+ + − − + = .
Gi¶i: XÐt hµm sè 2 2
1 1y x x x x= + + − − +
+ MiÒn x¸c ®Þnh D=R .
+ §¹o hµm
+ −
= −
+ + − +
= ⇔ − + + = + − +
− + >
⇔ 
− + + = + − +
2 2
2 2
2 2 2 2
2 1 2 1
'
2 1 2 1
' 0 (2 1) 1 (2 1) 1
(2 1)(2 1) 0
(vo nghiem)
(2 1) ( 1) (2 1) ( 1)
x x
y
x x x x
y x x x x x x
x x
x x x x x x
+ y’(0)=1>0 nªn hµm sè §B
2
+ Giíi h¹n
→−∞ →−∞
→+∞
= = −
+ + − − +
=
2 2
2
lim lim 1
1 1
lim 1.
x x
x
x
y
x x x x
y
+ BBT
x -∞ +∞
y’ +
y 1
-1
VËy ph¬ng tr×nh cã nghiÖm khi vµ chØ khi -1<m<1.
Bµi 4: T×m m ®Ó ph¬ng tr×nh sau cã nghiÖm thùc 2 1x x m+ = +
Gi¶i:
- §Æt 1; 0t x t= + ≥ . Ph¬ng tr×nh ®· cho trë thµnh:
2t=t2
-1+m m=-t2
+2t+1
- XÐt hµm sè y=-t2
+2t+1; t≥0; y’=-2t+2
x 0 1 +∞
y’ + 0 -
y 2
1 -∞
- Theo yªu cÇu cña bµi to¸n ®êng th¼ng y=m c¾t §THS khi m≤2.
Bµi 5: T×m m ®Ó ph¬ng tr×nh sau cã ®óng 2 nghiÖm d¬ng:
2 2
4 5 4x x m x x− + = + − .
Gi¶i:
- §Æt
2
2
2
( ) 4 5; '( ) ; '( ) 0 2
4 5
x
t f x x x f x f x x
x x
−
= = − + = = ⇔ =
− +
.
3
XÐt x>0 ta cã BBT:
x 0 2 +∞
f’(x) - 0 +
f(x) 5 +∞
1
- Khi ®ã ph¬ng tr×nh ®· cho trë thµnh m=t2
+t-5 t2
+t-5-m=0 (1).
- NÕu ph¬ng tr×nh (1) cã nghiÖm t1; t2 th× t1+ t2 =-1. Do ®ã (1) cã nhiÒu nhÊt 1
nghiÖm t≥1.
- VËy ph¬ng tr×nh ®· cho cã ®óng 2 nghiÖm d¬ng khi vµ chØ khi ph¬ng tr×nh (1)
cã ®óng 1 nghiÖm t (1; 5)∈ .
- §Æt g(t)=t2
+t-5. Ta ®i t×m m ®Ó ph¬ng tr×nh g(t)=m cã ®óng 1 nghiÖm t (1; 5)∈ .
f’(t)=2t+1>0 víi mäi t (1; 5)∈ . Ta cã BBT sau:
t 1 5
g’(t) +
g(t) 5
-3
Tõ BBT suy ra -3<m< 5 lµ c¸c gi¸ trÞ cÇn t×m.
Bµi 6: X¸c ®Þnh m ®Ó ph¬ng tr×nh sau cã nghiÖm
2 2 4 2 2
( 1 1 2) 2 1 1 1m x x x x x+ − − + = − + + − − .
Gi¶i:
- §iÒu kiÖn -1≤x≤1. §Æt 2 2
1 1t x x= + − − .
- Ta cã
2 2
2 4
1 1 0; 0 0
2 2 1 2 2; 2 1
x x t t x
t x t t x
+ ≥ − ⇒ ≥ = ⇔ =
= − − ≤ ⇒ ≤ = ⇔ = ±
- TËp gi¸ trÞ cña t lµ 0; 2 
  (t liªn tôc trªn ®o¹n [-1;1]). Ph¬ng tr×nh ®· cho trë thµnh:
2
2 2
( 2) 2 (*)
2
t t
m t t t m
t
− + +
+ = − + + ⇔ =
+
- XÐt
2
2
( ) ;0 2.
2
t t
f t t
t
− + +
= ≤ ≤
+
Ta cã f(t) liªn tôc trªn ®o¹n 0; 2 
  . Ph¬ng tr×nh ®· cho
cã nghiÖm x khi vµ chØ khi ph¬ng tr×nh (*) cã nghiÖm t thuéc 0; 2 
 
0; 2 0; 2
min ( ) max ( )f t m f t
   
   
⇔ ≤ ≤
.
- Ta cã
2
2
0; 2 0; 2
4
'( ) 0, 0; 2 ( ) 0; 2 .
( 2)
Suy ra min ( ) ( 2) 2 1;max ( ) (0) 1
t t
f t t f t NB
t
f t f f t f
   
   
− −    = ≤ ∀ ∈ ⇒
   +
= = − = = .
- VËy 2 1 1.m− ≤ ≤
4
Bài 7: Tìm m để bất phương trình 3 1mx x m− − ≤ + (1) có nghiệm.
Giải: Đặt 3; [0; )t x t= − ∈ +∞ . Bất phương trình trở thành:
2 2
2
1
( 3) 1 ( 2) 1
2
t
m t t m m t t m
t
+
+ − ≤ + ⇔ + ≤ + ⇔ ≤
+
(2)
(1)có nghiệm (2) có nghiệm t≥0  có ít nhất 1 điểm của ĐTHS y= 2
1
2
t
t
+
+
với t≥0 không ở
phía dưới đường thẳng y=m.
Xét y= 2
1
2
t
t
+
+
với t≥0 có
2
2 2
2 2
'
( 2)
t t
y
t
− − +
=
+
t 1 3− − 0 1 3− + +
∞
y’ - 0 + | + 0 -
y 3 1
4
+
Từ Bảng biến thiên ta có m≤
3 1
4
+
.
Bài 8: Tìm m để phương trình 3 6 (3 )(6 )x x x x m+ + − − + − = có nghiệm.
Giải:
Đặt ( ) 3 6t f x x x= = + + − với [ 3;6]x∈ − thì
6 3
' '( )
2 (6 )(3 )
x x
t f x
x x
− − +
= =
− +
x -3 3/2 6
+∞
f’(x) ║ + 0 - ║
f(x) | 3 2 |
3 3
Vậy t [3;3 2]∈ . Phương trình (1) trở thành
2 2
9 9
2 2 2
t t
t m t m
−
− = ⇔ − + + = (2).
Phương trình (1) có nghiệm Phương trình (2) có nghiệm t [3;3 2]∈  đường thẳng y=m có
điểm chung với đồ thị y=
2
9
2 2
t
t− + + với t [3;3 2]∈ .
Ta có y’=-t+1 nên có
t 1 3 3 2
y’ + 0 - | - |
y 3
9
3 2
2
−
5
Bài 9: Cho bất phương trình 21
(4 )(2 ) (18 2 )
4
x x a x x− + ≥ − + − . Tìm a để bất phương trình
nghiệm đúng với mọi x∈[-2;4].
Giải:
Đặt 2
(4 )(2 ) 2 8; [0;3]t x x x x t= − + = − + + ∈ . Bất phương trình trở thành:
2 21
(10 ) 4 10
4
t a t a t t≥ − + ⇔ ≥ − + .(2)
(1)ghiệm  (2) có nghiệm mọi t∈[0;3] đường thẳng y=a nằm trên ĐTHS
y=t2
-4t+10 với t∈[0;3]
y’=2t-4; y’=0t=2
t 0 2 3
y’ | - 0 + |
y 10 7
6
Vậy m≥10.
Bài 10: Cho phương trình 4 2 2 2
( 1)x x x m x+ + = + (1). Tìm m để phương trình có nghiệm.
Giải:
Phương trình đã cho tương đương
3 2 2 2
2
2 2 2 2 2 2
4( ) 4 ( 1) 4 2 2
4 2. ( ) 4
(1 ) (1 ) 1 1
x x x x x x x x
m m m
x x x x
+ + + +
= ⇔ = ⇔ + =
+ + + +
Đặt t= 2
2
1
x
x+
; t∈[-1;1].
Khi đó phương trình (1) trở thành 2t+t2
=4m.
(1) có nghiệm  (2) có nghiệm t∈[-1;1]
Xét hàm số y=f(t)=t2
+2t với t∈[-1;1]. Ta có f’(t)=2t+2≥0 với mọi t∈[-1;1].
t -1 1
f’ 0 + |
f 3
-1
Từ BBT -1≤4m≤3
1 3
4 4
m⇔ − ≤ ≤ .
6
HUYÊN ĐỀ : PHƯƠNG PHÁP GIẢI PHƯƠNG TRÌNH VÔ TỈ
I. PHƯƠNG PHÁP BIẾN ĐỔI TƯƠNG ĐƯƠNG
1. Bình phương 2 vế của phương trình
a) Phương pháp
 Thông thường nếu ta gặp phương trình dạng : A B C D+ = + , ta thường bình phương 2 vế ,
điều đó đôi khi lại gặp khó khăn hãy giải ví dụ sau
 ( )3 3 3 33 3
3 .A B C A B A B A B C+ = ⇒ + + + =
và ta sử dụng phép thế : 3 3
A B C+ = ta được phương trình : 3
3 . .A B A B C C+ + =
b) Ví dụ
Bài 1. Giải phương trình sau : 3 3 1 2 2 2x x x x+ + + = + +
Giải: Đk 0x ≥
Bình phương 2 vế không âm của phương trình ta được: ( ) ( ) ( )1 3 3 1 2 2 1x x x x x+ + + = + + , để giải
phương trình này dĩ nhiên là không khó nhưng hơi phức tạp một chút .
Phương trình giải sẽ rất đơn giản nếu ta chuyển vế phương trình : 3 1 2 2 4 3x x x x+ − + = − +
Bình phương hai vế ta có : 2 2
6 8 2 4 12 1x x x x x+ + = + ⇔ =
Thử lại x=1 thỏa
 Nhận xét : Nếu phương trình : ( ) ( ) ( ) ( )f x g x h x k x+ = +
Mà có : ( ) ( ) ( ) ( )f x h x g x k x+ = + , thì ta biến đổi phương trình về dạng :
( ) ( ) ( ) ( )f x h x k x g x− = − sau đó bình phương ,giải phương trình hệ quả
Bài 2. Giải phương trình sau :
3
21
1 1 3
3
x
x x x x
x
+
+ + = − + + +
+
Giải:
Điều kiện : 1x ≥ −
Bình phương 2 vế phương trình ?
Nếu chuyển vế thì chuyển như thế nào?
Ta có nhận xét :
3
21
. 3 1. 1
3
x
x x x x
x
+
+ = − + +
+
, từ nhận xét này ta có lời giải như sau :
3
21
(2) 3 1 1
3
x
x x x x
x
+
⇔ − + = − + − +
+
Bình phương 2 vế ta được:
3
2 2 1 31
1 2 2 0
3 1 3
xx
x x x x
x x
 = −+
= − − ⇔ − − = ⇔ 
+ = +
Thử lại : 1 3, 1 3x x= − = + l nghiệm
Qua lời giải trên ta có nhận xét : Nếu phương trình : ( ) ( ) ( ) ( )f x g x h x k x+ = +
Mà có : ( ) ( ) ( ) ( ). .f x h x k x g x= thì ta biến đổi ( ) ( ) ( ) ( )f x h x k x g x− = −
2. Trục căn thức
2.1. Trục căn thức để xuất hiện nhân tử chung
a) Phương pháp
Một số phương trình vô tỉ ta có thể nhẩm được nghiệm 0x như vậy phương trình luôn đưa về được
dạng tích ( ) ( )0 0x x A x− = ta có thể giải phương trình ( ) 0A x = hoặc chứng minh ( ) 0A x = vô nghiệm ,
chú ý điều kiện của nghiệm của phương trình để ta có thể đánh gía ( ) 0A x = vô nghiệm
7
b) Ví dụ
Bài 1 . Giải phương trình sau : ( )2 2 2 2
3 5 1 2 3 1 3 4x x x x x x x− + − − = − − − − +
Giải:
Ta nhận thấy : ( ) ( ) ( )2 2
3 5 1 3 3 3 2 2x x x x x− + − − − = − − v ( ) ( ) ( )2 2
2 3 4 3 2x x x x− − − + = −
Ta có thể trục căn thức 2 vế :
( ) 2 22 2
2 4 3 6
2 3 43 5 1 3 1
x x
x x xx x x x
− + −
=
− + − +− + + − +
Dể dàng nhận thấy x=2 là nghiệm duy nhất của phương trình .
Bài 2. Giải phương trình sau (OLYMPIC 30/4 đề nghị) : 2 2
12 5 3 5x x x+ + = + +
Giải: Để phương trình có nghiệm thì :
2 2 5
12 5 3 5 0
3
x x x x+ − + = − ≥ ⇔ ≥
Ta nhận thấy : x=2 là nghiệm của phương trình , như vậy phương trình có thể phân tích về dạng
( ) ( )2 0x A x− = , để thực hiện được điều đó ta phải nhóm , tách như sau :
( )
( )
2 2
2 2
2 2
2 2
4 4
12 4 3 6 5 3 3 2
12 4 5 3
2 1
2 3 0 2
12 4 5 3
x x
x x x x
x x
x x
x x
x x
− −
+ − = − + + − ⇔ = − +
+ + + +
 + +
⇔ − − − = ⇔ = ÷
+ + + + 
Dễ dàng chứng minh được : 2 2
2 2 5
3 0,
312 4 5 3
x x
x
x x
+ +
− − < ∀ >
+ + + +
Bài 3. Giải phương trình : 2 33
1 1x x x− + = −
Giải :Đk 3
2x ≥
Nhận thấy x=3 là nghiệm của phương trình , nên ta biến đổi phương trình
( )
( )
( ) ( )2
2 33
2 32 233
3 3 93
1 2 3 2 5 3 1
2 51 2 1 4
x x xx
x x x x
xx x
 
− + ++ 
− − + − = − − ⇔ − + = 
− +− + − +  
Ta chứng minh :
( ) ( )
222 2 23 33
3 3
1 1 2
1 2 1 4 1 1 3
x x
x x x
+ +
+ = + <
− + − + − + +
2
3
3 9
2 5
x x
x
+ +
<
− +
Vậy pt có nghiệm duy nhất x=3
2.2. Đưa về “hệ tạm “
a) Phương pháp
 Nếu phương trình vô tỉ có dạng A B C+ = , mà : A B Cα− =
ở dây C có thể là hàng số ,có thể là biểu thức của x. Ta có thể giải như sau :
A B
C A B
A B
α
−
= ⇒ − =
−
, khi đĩ ta có hệ: 2
A B C
A C
A B
α
α
 + =
⇒ = +
− =
b) Ví dụ
Bài 4. Giải phương trình sau : 2 2
2 9 2 1 4x x x x x+ + + − + = +
Giải:
Ta thấy : ( ) ( ) ( )2 2
2 9 2 1 2 4x x x x x+ + − − + = +
4x = − không phải là nghiệm
Xét 4x ≠ −
Trục căn thức ta có :
2 2
2 2
2 8
4 2 9 2 1 2
2 9 2 1
x
x x x x x
x x x x
+
= + ⇒ + + − − + =
+ + − − +
8
Vậy ta có hệ:
2 2
2
2 2
0
2 9 2 1 2
2 2 9 6 8
2 9 2 1 4 7
x
x x x x
x x x
xx x x x x
= + + − − + = ⇒ + + = + ⇔
 =+ + + − + = + 
Thử lại thỏa; vậy phương trình có 2 nghiệm : x=0 v x=
8
7
Bài 5. Giải phương trình : 2 2
2 1 1 3x x x x x+ + + − + =
Ta thấy : ( ) ( )2 2 2
2 1 1 2x x x x x x+ + − − + = + , như vậy không thỏa mãn điều kiện trên.
Ta có thể chia cả hai vế cho x và đặt
1
t
x
= thì bài toán trở nên đơn giản hơn
Bài tập đề nghị
Giải các phương trình sau :
( )2 2
3 1 3 1x x x x+ + = + +
4 3 10 3 2x x− − = − (HSG Toàn Quốc
2002)
( ) ( ) ( ) ( )2 2 5 2 10x x x x x− − = + − −
23
4 1 2 3x x x+ = − + −
2 33
1 3 2 3 2x x x− + − = −
2 3
2 11 21 3 4 4 0x x x− + − − = (OLYMPIC 30/4-2007)
2 2 2 2
2 1 3 2 2 2 3 2x x x x x x x− + − − = + + + − +
2 2
2 16 18 1 2 4x x x x+ + + − = +
2 2
15 3 2 8x x x+ = − + +
3. Phương trình biến đổi về tích
 Sử dụng đẳng thức
( ) ( )1 1 1 0u v uv u v+ = + ⇔ − − =
( ) ( ) 0au bv ab vu u b v a+ = + ⇔ − − =
2 2
A B=
Bài 1. Giải phương trình : 233 3
1 2 1 3 2x x x x+ + + = + + +
Giải: ( )( )3 3
0
1 1 2 1 0
1
x
pt x x
x
=
⇔ + − + − = ⇔  = −
Bi 2. Giải phương trình : 2 23 33 3
1x x x x x+ + = + +
Giải:
+ 0x = , không phải là nghiệm
+ 0x ≠ , ta chia hai vế cho x: ( )3 3 33 3
1 1
1 1 1 1 0 1
x x
x x x x
x x
 + +
+ = + + ⇔ − − = ⇔ = ÷
 
Bài 3. Giải phương trình: 2
3 2 1 2 4 3x x x x x x+ + + = + + +
Giải: : 1dk x ≥ −
pt ( )( )
1
3 2 1 1 0
0
x
x x x
x
=
⇔ + − + − = ⇔  =
Bài 4. Giải phương trình :
4
3 4
3
x
x x
x
+ + =
+
Giải:
Đk: 0x ≥
Chia cả hai vế cho 3x + :
2
4 4 4
1 2 1 0 1
3 3 3
x x x
x
x x x
 
+ = ⇔ − = ⇔ = ÷
+ + + 
 Dùng hằng đẳng thức
9
Biến đổi phương trình về dạng : k k
A B=
Bài 1. Giải phương trình : 3 3x x x− = +
Giải:
Đk: 0 3x≤ ≤ khi đó pt đ cho tương đương : 3 2
3 3 0x x x+ + − =
3 3
1 10 10 1
3 3 3 3
x x
− 
⇔ + = ⇔ = ÷
 
Bài 2. Giải phương trình sau : 2
2 3 9 4x x x+ = − −
Giải:
Đk: 3x ≥ − phương trình tương đương : ( )
2
2
1
3 1 3
1 3 9 5 97
3 1 3
18
x
x x
x x
xx x
= + + = + + = ⇔ ⇔ − − =+ + = − 
Bài 3. Giải phương trình sau : ( ) ( )
22 332 3 9 2 2 3 3 2x x x x x+ + = + +
Giải : pttt ( )
3
3 3
2 3 0 1x x x⇔ + − = ⇔ =
II. PHƯƠNG PHÁP ĐẶT ẦN PHỤ
1. Phương pháp đặt ẩn phụ thông thường
 Đối với nhiều phương trình vô vô tỉ , để giải chúng ta có thể đặt ( )t f x= và chú ý điều kiện của t
nếu phương trình ban đầu trở thành phương trình chứa một biến t quan trọng hơn ta có thể giải được
phương trình đó theo t thì việc đặt phụ xem như “hoàn toàn ” .Nói chung những phương trình mà có thể
đặt hoàn toàn ( )t f x= thường là những phương trình dễ .
Bài 1. Giải phương trình: 2 2
1 1 2x x x x− − + + − =
Điều kiện: 1x ≥
Nhận xét. 2 2
1. 1 1x x x x− − + − =
Đặt 2
1t x x= − − thì phương trình có dạng:
1
2 1t t
t
+ = ⇔ =
Thay vào tìm được 1x =
Bài 2. Giải phương trình: 2
2 6 1 4 5x x x− − = +
Giải
Điều kiện:
4
5
x ≥ −
Đặt 4 5( 0)t x t= + ≥ thì
2
5
4
t
x
−
= . Thay vào ta có phương trình sau:
4 2
2 4 210 25 6
2. ( 5) 1 22 8 27 0
16 4
t t
t t t t t
− +
− − − = ⇔ − − + =
2 2
( 2 7)( 2 11) 0t t t t⇔ + − − − =
Ta tìm được bốn nghiệm là: 1,2 3,41 2 2; 1 2 3t t= − ± = ±
Do 0t ≥ nên chỉ nhận các gái trị 1 31 2 2, 1 2 3t t= − + = +
Từ đó tìm được các nghiệm của phương trình l: 1 2 2 3vaøx x= − = +
Cách khác: Ta có thể bình phương hai vế của phương trình với điều kiện 2
2 6 1 0x x− − ≥
Ta được: 2 2 2
( 3) ( 1) 0x x x− − − = , từ đó ta tìm được nghiệm tương ứng.
Đơn giản nhất là ta đặt : 2 3 4 5y x− = + và đưa về hệ đối xứng (Xem phần dặt ẩn phụ đưa về hệ)
Bài 3. Giải phương trình sau: 5 1 6x x+ + − =
10
Điều kiện: 1 6x≤ ≤
Đặt 1( 0)y x y= − ≥ thì phương trình trở thnh: 2 4 2
5 5 10 20 0y y y y y+ + = ⇔ − − + = ( với
5)y ≤ 2 2
( 4)( 5) 0y y y y⇔ + − − − =
1 21 1 17
,
2 2
(loaïi)y y
+ − +
⇔ = =
Từ đó ta tìm được các giá trị của
11 17
2
x
−
=
Bài 4. (THTT 3-2005) Giải phương trình sau : ( )( )
2
2004 1 1x x x= + − −
Giải: đk 0 1x≤ ≤
Đặt 1y x= − pttt ( ) ( )2 2
2 1 1002 0 1 0y y y y x⇔ − + − = ⇔ = ⇔ =
Bài 5. Giải phương trình sau : 2 1
2 3 1x x x x
x
+ − = +
Giải:
Điều kiện: 1 0x− ≤ <
Chia cả hai vế cho x ta nhận được:
1 1
2 3x x
x x
+ − = +
Đặt
1
t x
x
= − , ta giải được.
Bài 6. Giải phương trình : 2 4 23
2 1x x x x+ − = +
Giải: 0x = không phải là nghiệm , Chia cả hai vế cho x ta được: 3
1 1
2x x
x x
 
− + − = ÷
 
Đặt t= 3
1
x
x
− , Ta có : 3
2 0t t+ − = ⇔
1 5
1
2
t x
±
= ⇔ =
Bài tập đề nghị
Giải các phương trình sau
2 2
15 2 5 2 15 11x x x x− − = − +
2
( 5)(2 ) 3 3x x x x+ − = +
2
(1 )(2 ) 1 2 2x x x x+ − = + −
2 2
17 17 9x x x x+ − + − =
2
3 2 1 4 9 2 3 5 2x x x x x− + − = − + − +
2 2
11 31x x+ + =
2 2 2
2 (1 ) 3 1 (1 ) 0nn n
x x x+ + − + − =
2
(2004 )(1 1 )x x x= + − −
( 3 2)( 9 18) 168x x x x x+ + + + =
32 2
1 2 1 3x x− + − =
Nhận xét : đối với cách đặt ẩn phụ như trên chúng ta chỉ giải quyết được một lớp bài đơn giản, đôi khi
phương trình đối với t lại quá khó giải
2. Đặt ẩn phụ đưa về phương trình thuần nhất bậc 2 đối với 2 biến :
 Chúng ta đã biết cách giải phương trình:
2 2
0u uv vα β+ + = (1) bằng cách
Xét 0v ≠ phương trình trở thành :
2
0
u u
v v
α β
   
+ + = ÷  ÷
   
0v = thử trực tiếp
Các trường hợp sau cũng đưa về được (1)
 ( ) ( ) ( ) ( ). .a A x bB x c A x B x+ =
 2 2
u v mu nvα β+ = +
11
Chúng ta hãy thay các biểu thức A(x) , B(x) bởi các biểu thức vô tỉ thì sẽ nhận được phương trình vô tỉ theo
dạng này .
a) . Phương trình dạng : ( ) ( ) ( ) ( ). .a A x bB x c A x B x+ =
Như vậy phương trình ( ) ( )Q x P xα= có thể giải bằng phương pháp trên nếu
( ) ( ) ( )
( ) ( ) ( )
.P x A x B x
Q x aA x bB x
 =

= +
Xuất phát từ đẳng thức :
( ) ( )3 2
1 1 1x x x x+ = + − +
( ) ( )( )4 2 4 2 2 2 2
1 2 1 1 1x x x x x x x x x+ + = + + − = + + − +
( )( )4 2 2
1 2 1 2 1x x x x x+ = − + + +
( )( )4 2 2
4 1 2 2 1 2 2 1x x x x x+ = − + + +
Hãy tạo ra những phương trình vô tỉ dạng trên ví dụ như: 2 4
4 2 2 4 1x x x− + = +
Để có một phương trình đẹp , chúng ta phải chọn hệ số a,b,c sao cho phương trình bậc hai 2
0at bt c+ − =
giải “ nghiệm đẹp”
Bài 1. Giải phương trình : ( )2 3
2 2 5 1x x+ = +
Giải: Đặt 2
1, 1u x v x x= + = − +
Phương trình trở thành : ( )2 2
2
2 5 1
2
u v
u v uv
u v
=
+ = ⇔
 =

Tìm được:
5 37
2
x
±
=
Bài 2. Giải phương trình : 2 4 23
3 1 1
3
x x x x− + = − + +
Bài 3: giải phương trình sau : 2 3
2 5 1 7 1x x x+ − = −
Giải:
Đk: 1x ≥
Nhận xt : Ta viết ( ) ( ) ( ) ( )2 2
1 1 7 1 1x x x x x xα β− + + + = − + +
Đồng nhất thức ta được: ( ) ( ) ( ) ( )2 2
3 1 2 1 7 1 1x x x x x x− + + + = − + +
Đặt
2
1 0, 1 0u x v x x= − ≥ = + + > , ta được:
9
3 2 7 1
4
v u
u v uv
v u
=
+ = ⇔
 =

Ta được : 4 6x = ±
Bài 4. Giải phương trình : ( )
33 2
3 2 2 6 0x x x x− + + − =
Giải:
Nhận xét : Đặt 2y x= + ta hãy biến pt trên về phương trình thuần nhất bậc 3 đối với x và y :
3 2 3 3 2 3
3 2 6 0 3 2 0
2
x y
x x y x x xy y
x y
=
− + − = ⇔ − + = ⇔  = −
Pt có nghiệm : 2, 2 2 3x x= = −
b).Phương trình dạng : 2 2
u v mu nvα β+ = +
Phương trình cho ở dạng này thường khó “phát hiện “ hơn dạng trên , nhưg nếu ta bình phương hai vế thì
đưa về được dạng trên.
Bài 1. giải phương trình : 2 2 4 2
3 1 1x x x x+ − = − +
Giải:
12
Ta đặt :
2
2
1
u x
v x
 =

= −
khi đó phương trình trở thành : 2 2
3u v u v+ = −
Bài 2.Giải phương trình sau : 2 2
2 2 1 3 4 1x x x x x+ + − = + +
Giải
Đk
1
2
x ≥ . Bình phương 2 vế ta có :
( )( ) ( )( ) ( ) ( )2 2 2 2
2 2 1 1 2 2 1 2 2 1x x x x x x x x x x+ − = + ⇔ + − = + − −
Ta có thể đặt :
2
2
2 1
u x x
v x
 = +

= −
khi đó ta có hệ :
2 2
1 5
2
1 5
2
u v
uv u v
u v
 −
=
= − ⇔
 +
=

Do , 0u v ≥ . ( )21 5 1 5
2 2 1
2 2
u v x x x
+ +
= ⇔ + = −
Bài 3. giải phương trình : 2 2
5 14 9 20 5 1x x x x x− + − − − = +
Giải:
Đk 5x ≥ . Chuyển vế bình phương ta được: ( )( )2 2
2 5 2 5 20 1x x x x x− + = − − +
Nhận xét : không tồn tại số ,α β để : ( ) ( )2 2
2 5 2 20 1x x x x xα β− + = − − + + vậy ta không thể đặt
2
20
1
u x x
v x
 = − −

= +
.
Nhưng may mắn ta có : ( )( ) ( ) ( ) ( ) ( ) ( )2 2
20 1 4 5 1 4 4 5x x x x x x x x x− − + = + − + = + − −
Ta viết lại phương trình: ( ) ( )2 2
2 4 5 3 4 5 ( 4 5)( 4)x x x x x x− − + + = − − + . Đến đây bài toán được giải
quyết .
Các em hãy tự sáng tạo cho mình những phương trình vô tỉ “đẹp “ theo cách trên
3. Phương pháp đặt ẩn phụ không hoàn toàn
 Từ những phương trình tích ( )( )1 1 1 2 0x x x+ − + − + = ,( )( )2 3 2 3 2 0x x x x+ − + − + =
Khai triển và rút gọn ta sẽ được những phương trình vô tỉ không tầm thường chút nào, độ khó của phương
trình dạng này phụ thuộc vào phương trình tích mà ta xuất phát .
Từ đó chúng ta mới đi tìm cách giải phương trình dạng này .Phương pháp giải được thể hiện qua các ví dụ
sau .
Bài 1. Giải phương trình : ( )2 2 2
3 2 1 2 2x x x x+ − + = + +
Giải:
2
2t x= + , ta có : ( )2 3
2 3 3 0
1
t
t x t x
t x
=
− + − + = ⇔  = −
Bài 2. Giải phương trình : ( ) 2 2
1 2 3 1x x x x+ − + = +
Giải:
Đặt : 2
2 3, 2t x x t= − + ≥ Khi đó phương trình trở thnh : ( ) 2
1 1x t x+ = +
( )2
1 1 0x x t⇔ + − + =
13
Bây giờ ta thêm bớt , để được phương trình bậc 2 theo t có ∆ chẵn :
( ) ( ) ( ) ( )2 2 2
2 3 1 2 1 0 1 2 1 0
1
t
x x x t x t x t x
t x
=
− + − + + − = ⇔ − + + − = ⇔  = −
Từ một phương trình đơn giản : ( )( )1 2 1 1 2 1 0x x x x− − + − − + + = , khai triển ra ta sẽ được pt
sau
Bài 3. Giải phương trình sau : 2
4 1 1 3 2 1 1x x x x+ − = + − + −
Giải:
Nhận xét : đặt 1t x= − , pttt: 4 1 3 2 1x x t t x+ = + + + (1)
Ta rút 2
1x t= − thay vào thì được pt: ( ) ( )2
3 2 1 4 1 1 0t x t x− + + + + − =
Nhưng không có sự may mắn để giải được phương trình theo t ( ) ( )
2
2 1 48 1 1x x∆ = + + − + − không
có dạng bình phương .
Muốn đạt được mục đích trên thì ta phải tách 3x theo ( ) ( )
2 2
1 , 1x x− +
Cụ thể như sau : ( ) ( )3 1 2 1x x x= − − + + thay vào pt (1) ta được:
Bài 4. Giải phương trình: 2
2 2 4 4 2 9 16x x x+ + − = +
Giải .
Bình phương 2 vế phương trình: ( ) ( ) ( )2 2
4 2 4 16 2 4 16 2 9 16x x x x+ + − + − = +
Ta đặt : ( )2
2 4 0t x= − ≥ . Ta được: 2
9 16 32 8 0x t x− − + =
Ta phải tách ( ) ( )2 2 2
9 2 4 9 2 8x x xα α α= − + + − làm sao cho t∆ có dạng chính phương .
Nhận xét : Thông thường ta chỉ cần nhóm sao cho hết hệ số tự do thì sẽ đạt được mục đích
4. Đặt nhiều ẩn phụ đưa về tích
 Xuất phát từ một số hệ “đại số “ đẹp chúng ta có thể tạo ra được những phương trình vô tỉ mà khi giải
nó chúng ta lại đặt nhiều ẩn phụ và tìm mối quan hệ giữa các ẩn phụ để đưa về hệ
Xuất phát từ đẳng thức ( ) ( ) ( ) ( )
3 3 3 3
3a b c a b c a b b c c a+ + = + + + + + + , Ta có
( ) ( )( ) ( )
33 3 3
0a b c a b c a b a c b c+ + = + + ⇔ + + + =
Từ nhận xét này ta có thể tạo ra những phương trình vô tỉ có chứa căn bậc ba .
2 23 33
7 1 8 8 1 2x x x x x+ − − − + − + =
3 3 3 3
3 1 5 2 9 4 3 0x x x x+ + − + − − − =
Bài 1. Giải phương trình : 2 . 3 3 . 5 5 . 2x x x x x x x= − − + − − + − −
Giải :
2
3
5
u x
v x
w x
 = −

= −

= −
, ta có :
( )( )
( )( )
( )( )
2
2
2
22
3 3
5 5
u v u wu uv vw wu
v uv vw wu u v v w
w uv vw wu v w u w
 + + = − = + +

− = + + ⇔ + + = 
 
− = + + + + = 
, giải hệ ta được:
30 239
60 120
u x= ⇔ =
Bài 2. Giải phương trình sau : 2 2 2 2
2 1 3 2 2 2 3 2x x x x x x x− + − − = + + + − +
14
Giải . Ta đặt :
2
2
2
2
2 1
3 2
2 2 3
2
a x
b x x
c x x
d x x
 = −

 = − −

= + +

= − +
, khi đó ta có : 2 2 2 2
2
a b c d
x
a b c d
+ = +
⇔ = −
− = −
Bài 3. Giải các phương trình sau
1) 2 2
4 5 1 2 1 9 3x x x x x+ + − − + = −
2) ( ) ( ) ( )
3 3 244 44 1 1 1 1x x x x x x x x+ − + − = − + + −
5. Đặt ẩn phụ đưa về hệ:
5.1 Đặt ẩn phụ đưa về hệ thông thường
 Đặt ( ) ( ),u x v xα β= = và tìm mối quan hệ giữa ( )xα và ( )xβ từ đó tìm được hệ theo u,v
Bài 1. Giải phương trình: ( )3 33 3
25 25 30x x x x− + − =
Đặt 3 3 3 3
35 35y x x y= − ⇒ + =
Khi đó phương trình chuyển về hệ phương trình sau: 3 3
( ) 30
35
xy x y
x y
+ =

+ =
, giải hệ này ta tìm được
( ; ) (2;3) (3;2)x y = = . Tức là nghiệm của phương trình là {2;3}x∈
Bài 2. Giải phương trình:
4
4
1
2 1
2
x x− − + =
Điều kiện: 0 2 1x≤ ≤ −
Đặt
4
4
2 1
0 2 1,0 2 1
x u
u v
x v
 − − =
⇒ ≤ ≤ − ≤ ≤ −
=
Ta đưa về hệ phương trình sau:
4
4
2
2 4 4
4
1
1
2
2
1
2 1 2 1
2
u v
u v
u v v v

= − + = 
⇔ 
  + = − − + = −  ÷ 
Giải phương trình thứ 2:
2
2 2
4
1
( 1) 0
2
v v
 
+ − + = ÷
 
, từ đó tìm ra v rồi thay vào tìm nghiệm của phương
trình.
Bài 3. Giải phương trình sau: 5 1 6x x+ + − =
Điều kiện: 1x ≥
Đặt 1, 5 1( 0, 0)a x b x a b= − = + − ≥ ≥ thì ta đưa về hệ phương trình sau:
2
2
5
( )( 1) 0 1 0 1
5
a b
a b a b a b a b
b a
 + =
→ + − + = ⇒ − + = ⇒ = −
− =
Vậy
11 17
1 1 5 1 1 5
2
x x x x x
−
− + = + − ⇔ − = − ⇒ =
Bài 8. Giải phương trình:
6 2 6 2 8
35 5
x x
x x
− +
+ =
− +
Giải
Điều kiện: 5 5x− < <
15
Đặt ( )5 , 5 0 , 10u x v y u v= − = − < < .
Khi đó ta được hệ phương trình:
22 2 ( ) 10 210
2 44 4 8
( ) 12( )
33
u v uvu v
u vu z
uvu v
 + = ++ =
 
⇔   
+ − =− − + + =  ÷ 
  
5.2 Xây dựng phương trình vô tỉ từ hệ đối xứng loại II
 Ta hãy đi tìm nguồn gốc của những bài toán giải phương trình bằng cách đưa về hệ đối xứng loại II
 Ta xét một hệ phương trình đối xứng loại II sau :
( )
( )
2
2
1 2 (1)
1 2 (2)
x y
y x
 + = +

+ = +
việc giải hệ này thì đơn
giản
Bây giời ta sẽ biến hệ thành phương trình bằng cách đặt ( )y f x= sao cho (2) luôn đúng , 2 1y x= + −
, khi đó ta có phương trình : ( )
2 2
1 ( 2 1) 1 2 2x x x x x+ = + − + ⇔ + = +
Vậy để giải phương trình : 2
2 2x x x+ = + ta đặt lại như trên và đưa về hệ
Bằng cách tương tự xét hệ tổng quát dạng bậc 2 :
( )
( )
2
2
x ay b
y ax b
α β
α β
 + = +

+ = +
, ta sẽ xây dựng được phương trình
dạng sau : đặt y ax bα β+ = + , khi đó ta có phương trình : ( )
2 a
x ax b b
β
α β
α α
+ = + + −
Tương tự cho bậc cao hơn : ( )
n na
x ax b b
β
α β
α α
+ = + + −
Tóm lại phương trình thường cho dưới dạng khai triển ta phải viết về dạng : ( ) ' '
n n
x p a x bα β γ+ = + +
v đặt n
y ax bα β+ = + để đưa về hệ , chú ý về dấu của α ???
Việc chọn ;α β thông thường chúng ta chỉ cần viết dưới dạng :( ) ' '
n n
x p a x bα β γ+ = + + là chọn được.
Bài 1. Giải phương trình: 2
2 2 2 1x x x− = −
Điều kiện:
1
2
x ≥
Ta có phương trình được viết lại là: 2
( 1) 1 2 2 1x x− − = −
Đặt 1 2 1y x− = − thì ta đưa về hệ sau:
2
2
2 2( 1)
2 2( 1)
x x y
y y x
 − = −

− = −
Trừ hai vế của phương trình ta được ( )( ) 0x y x y− + =
Giải ra ta tìm được nghiệm của phương trình là: 2 2x = +
Bài 6. Giải phương trình: 2
2 6 1 4 5x x x− − = +
Giải
Điều kiện
5
4
x ≥ −
Ta biến đổi phương trình như sau: 2 2
4 12 2 2 4 5 (2 3) 2 4 5 11x x x x x− − = + ⇔ − = + +
Đặt 2 3 4 5y x− = + ta được hệ phương trình sau:
2
2
(2 3) 4 5
( )( 1) 0
(2 3) 4 5
x y
x y x y
y x
 − = +
⇒ − + − =
− = +
Với 2 3 4 5 2 3x y x x x= ⇒ − = + ⇒ = +
Với 1 0 1 1 2x y y x x+ − = ⇒ = − → = −
16
Kết luận: Nghiệm của phương trình là {1 2; 1 3}− +
Các em hãy xây dựng một sồ hệ dạng này ?
 Dạng hệ gần đối xứng
Ta xt hệ sau :
2
2
(2 3) 2 1
(1)
(2 3) 3 1
x y x
y x
 − = + +

− = +
đây không phải là hệ đối xứng loại 2 nhưng chúng ta vẫn giải
hệ được , và từ hệ này chúng ta xây dưng được bài toán phương trình sau :
Bài 1 . Giải phương trình: 2
4 5 13 3 1 0x x x+ − + + =
Nhận xét : Nếu chúng ta nhóm như những phương trình trước :
2
13 33
2 3 1
4 4
x x
 
− = + − ÷
 
Đặt
13
2 3 1
4
y x− = + thì chúng ta không thu được hệ phương trình mà chúng ta có thể giải được.
Để thu được hệ (1) ta đặt : 3 1y xα β+ = + , chọn ,α β sao cho hệ chúng ta có thể giải được , (đối
xứng hoặc gần đối xứng )
Ta có hệ :
( )
2 2 2 2
22
2 3 1 0 (1)3 1
(*)
4 13 5 0 (2)4 13 5
y y xy x
x x yx x y
α αβ βα β
α βα β
  + − + − =+ = + 
⇔ 
− + + + =− + = − − 
Để giải hệ trên thì ta lấy (1) nhân với k cộng với (2): và mong muốn của chúng ta là có nghiệm x y=
Nên ta phải có :
2 2
2 3 1
4 13 5
α αβ β
α β
− −
= =
− +
, ta chọn được ngay 2; 3α β= − =
Ta có lời giải như sau :
Điều kiện:
1
3
x ≥ − , Đặt
3
3 1 (2 3), ( )
2
x y y+ = − − ≤
Ta có hệ phương trình sau:
2
2
(2 3) 2 1
( )(2 2 5) 0
(2 3) 3 1
x y x
x y x y
y x
 − = + +
⇒ − + − =
− = +
Với
15 97
8
x y x
−
= ⇒ =
Với
11 73
2 2 5 0
8
x y x
+
+ − = ⇒ =
Kết luận: tập nghiệm của phương trình là:
15 97 11 73
;
8 8
 − + 
 
  
Chú ý : khi đã làm quen, chúng ta có thể tìm ngay ;α β bằng cách viết lại phương trình
ta viết lại phương trình như sau: 2
(2 3) 3 1 4x x x− = − + + +
khi đó đặt 3 1 2 3x y+ = − + , nếu đặt 2 3 3 1y x− = + thì chúng ta không thu được hệ như mong
muốn , ta thấy dấu của α cùng dấu với dấu trước căn.
Một cách tổng quát .
Xét hệ:
( ) . . (1)
( ) '. ' (2)
f x A x B y m
f y A x m
= + +

= +
để hệ có nghiệm x = y thì : A-A’=B và m=m’,
Nếu từ (2) tìm được hàm ngược ( )y g x= thay vào (1) ta được phương trình
Như vậy để xây dựng pt theo lối này ta cần xem xét để có hàm ngược và tìm được và hơn nữa hệ phải giải
được.
Một số phương trình được xây dựng từ hệ.
Giải các phương trình sau
1) 2
4 13 5 3 1 0x x x− + + + = 4) 33
6 1 8 4 1x x x+ = − −
17
2) 2
4 13 5 3 1 0x x x− + + + =
3)
3 23 4
81 8 2 2
3
x x x x− = − + −
5) ( ) ( )215
30 4 2004 30060 1 1
2
x x x− = + +
6) 3 23
3 5 8 36 53 25x x x− = − + −
Giải (3):
Phương trình : ( )
33 23 3
27 81 8 27 54 36 54 27 81 8 3 2 46x x x x x x⇔ − = − + − ⇔ − = − −
Ta đặt : 3
3 2 81 8y x− = −
Các em hãy xây dựng những phương trình dạng này !
III. PHƯƠNG PHÁP ĐÁNH GIÁ
1. Dùng hằng đẳng thức :
 Từ những đánh giá bình phương : 2 2
0A B+ ≥ , ta xây dựng phương trình dạng 2 2
0A B+ =
Từ phương trình ( ) ( )
2 2
5 1 2 9 5 2 1 0x x x x− − + − − + − = ta khai triển ra có phương trình :
( )2
4 12 1 4 5 1 9 5x x x x x+ + − = − + −
2. Dùng bất đẳng thức
 Một số phương trình được tạo ra từ dấu bằng của bất đẳng thức:
A m
B m
≥

≤
nếu dấu bằng ỏ (1) và (2) cùng
dạt được tại 0x thì 0x là nghiệm của phương trình A B=
Ta có : 1 1 2x x+ + − ≤ Dấu bằng khi và chỉ khi 0x = và
1
1 2
1
x
x
+ + ≥
+
, dấu bằng khi và chỉ
khi x=0. Vậy ta có phương trình:
1
1 2008 1 2008 1
1
x x x
x
− + + = + +
+
Đôi khi một số phương trình được tạo ra từ ý tưởng :
( )
( )
A f x
B f x
 ≥

≤
khi đó :
( )
( )
A f x
A B
B f x
 =
= ⇔ 
=
 Nếu ta đoán trước được nghiệm thì việc dùng bất đẳng thức dễ dàng hơn, nhưng có nhiều bài
nghiệm là vô tỉ việc đoán nghiệm không được, ta vẫn dùng bất đẳng thức để đánh giá được
Bài 1. Giải phương trình (OLYMPIC 30/4 -2007):
2 2
9
1
x x
x
+ = +
+
Giải: Đk 0x ≥
Ta có : ( )
2 2
22 2 1
2 2 1 9
11 1
x
x x x
xx x
       + ≤ + + + = + ÷  ÷   ++ +     
Dấu bằng
2 2 1 1
71 1
x
x x
⇔ = ⇔ =
+ +
Bài 2. Giải phương trình : 2 4 2 4
13 9 16x x x x− + + =
Giải: Đk: 1 1x− ≤ ≤
Biến đổi pt ta có : ( )
2
2 2 2
13 1 9 1 256x x x− + + =
Áp dụng bất đẳng thức Bunhiacopxki:
( ) ( ) ( ) ( )
2
2 2 2 2 2
13. 13. 1 3. 3. 3 1 13 27 13 13 3 3 40 16 10x x x x x− + + ≤ + − + + = −
18
Áp dụng bất đẳng thức Côsi: ( )
2
2 2 16
10 16 10 64
2
x x
 
− ≤ = ÷
 
Dấu bằng
2
2
2 2
2
1
51
3
2
10 16 10
5
xx
x
xx x

 =+ − =
⇔ ⇔
 = −= − 
Bài 3. giải phương trình: 3` 2 4
3 8 40 8 4 4 0x x x x− − + − + =
Ta chứng minh : 4
8 4 4 13x x+ ≤ + và ( ) ( )
23 2
3 8 40 0 3 3 13x x x x x x− − + ≥ ⇔ − + ≥ +
Bài tập đề nghị .
Giải các phương trình sau
1 2 1 2
1 2 1 2
1 2 1 2
x x
x x
x x
− +
− + + = +
+ −
4 4 4
1 1 2 8x x x x+ − + − − = +
4 4 4
2 8 4 4 4 4x x x+ = + + −
4 33
16 5 6 4x x x+ = +
3` 2 4
3 8 40 8 4 4 0x x x x− − + − + =
3 3 4 2
8 64 8 28x x x x+ + − = − +
2
2
1 1
2 2 4x x
x x
 
− + − = − + ÷
 
3. Xây dựng bài toán từ tính chất cực trị hình học
3.1 Dùng tọa độ của véc tơ
 Trong mặt phẳng tọa độ Oxy, Cho các véc tơ: ( ) ( )1 1 2 2; , ;u x y v x y= =
r r
khi đó ta có
 ( ) ( )
2 2 2 2 2 2
1 2 1 2 1 1 2 2u v u v x x y y x y x y+ ≤ + ⇔ + + + ≤ + + +
r r r r
Dấu bằng xẩy ra khi và chỉ khi hai véc tơ ,u v
r r
cùng hướng
1 1
2 2
0
x y
k
x y
⇔ = = ≥ , chú ý tỉ số phải dương
 . . .cos .u v u v u vα= ≤
r r r r r r
, dấu bằng xẩy ra khi và chỉ khi cos 1 u vα = ⇔ ↑↑
r
3.2 Sử dụng tính chất đặc biệt về tam giác
 Nếu tam giác ABC là tam giác đều , thì với mọi điểm M trên mặt phẳng tam giác, ta luôn có
MA MB MC OA OB OC+ + ≥ + + với O là tâm của đường tròn .Dấu bằng xẩy ra khi và chỉ khi M O≡ .
 Cho tam giác ABC có ba góc nhọn và điểm M tùy ý trong mặt mặt phẳng Thì MA+MB+MC nhỏ
nhất khi điểm M nhìn các cạnh AB,BC,AC dưới cùng một góc 0
120
Bài tập
1) ( ) ( )2 2 2
2 2 1 2 3 1 1 2 3 1 1 3x x x x x x− + + − − + + + + + =
2)
2 2
4 5 10 50 5x x x x− + − − + =
IV. PHƯƠNG PHÁP HÀM SỐ
1.Xây dựng phương trình vô tỉ dựa theo hàm đơn điệu
 Dựa vào kết quả : “ Nếu ( )y f t= là hàm đơn điệu thì ( ) ( )f x f t x t= ⇔ = ” ta có thể xây dựng được
những phương trình vô tỉ
Xuất phát từ hàm đơn điệu : ( ) 3 2
2 1y f x x x= = + + mọi 0x ≥ ta xây dựng phương trình :
( ) ( ) ( )
3
3 2 2
3 1 2 1 2 3 1 (3 1) 1f x f x x x x x= − ⇔ + + = − + − + , Rút gọn ta được phương trình
19
( )3 2
2 3 1 2 3 1 3 1x x x x x+ − + = − −
Từ phương trình ( ) ( )1 3 1f x f x+ = − thì bài toán sẽ khó hơn ( ) ( )3 2
2 7 5 4 2 3 1 3 1x x x x x+ + + = − −
Để gải hai bài toán trên chúng ta có thể làm như sau :
Đặt 3 1y x= − khi đó ta có hệ :
3 2 3
2
2 7 5 4 2
3 1
x x x y
x y
 + + + =

− =
cộng hai phương trình ta được:
( ) ( )
3 2
2 1 1x x+ + + = 3 2
2y y+
Hãy xây dựng những hàm đơn điệu và những bài toán vô tỉ theo dạng trên ?
Bài 1. Giải phương trình : ( ) ( ) ( )2 2
2 1 2 4 4 4 3 2 9 3 0x x x x x+ + + + + + + =
Giải:
( ) ( )( ) ( ) ( )( ) ( ) ( )
2 2
2 1 2 2 1 3 3 2 3 3 2 1 3x x x x f x f x⇔ + + + + = − + − + ⇔ + = −
Xét hàm số ( ) ( )2
2 3f t t t= + + , là hàm đồng biến trên R, ta có
1
5
x = −
Bài 2. Giải phương trình 3 2 23
4 5 6 7 9 4x x x x x− − + = + −
Giải . Đặt 23
7 9 4y x x= + − , ta có hệ : ( ) ( )
3 2
33
2 3
4 5 6
1 1
7 9 4
x x x y
y y x x
x x y
 − − + =
⇒ + = + + +
+ − =
Xét hàm số : ( ) 3
f t t t= + , là hàm đơn điệu tăng. Từ phương trình
( ) ( ) ( ) 23
5
1 1 1 7 9 4 1 5
2
x
f y f x y x x x x
x
=
=  +  ⇔ = + ⇔ + = + − ⇔ − ±   =

Bài 3. Giải phương trình : 33
6 1 8 4 1x x x+ = − −
V. PHƯƠNG PHÁP LƯỢNG GIÁC HÓA
1. Một số kiến thức cơ bản:
 Nếu 1x ≤ thì có một số t với ;
2 2
t
π π− − 
∈  
sao cho : sint x= và một số y với [ ]0;y π∈ sao
cho cosx y=
 Nếu 0 1x≤ ≤ thì có một số t với 0;
2
t
π 
∈  
sao cho : sint x= và một số y với 0;
2
y
π 
∈   
sao
cho cosx y=
 Với mỗi số thực x có ;
2 2
t
π π 
∈ − ÷
 
sao cho : tanx t=
 Nếu : x, y là hai số thực thỏa:
2 2
1x y+ = , thì có một số t với 0 2t π≤ ≤ , sao cho
sin , cosx t y t= =
Từ đó chúng ta có phương pháp giải toán :
 Nếu : 1x ≤ thì đặt sint x= với ;
2 2
t
π π− − 
∈  
hoặc cosx y= với [ ]0;y π∈
 Nếu 0 1x≤ ≤ thì đặt sint x= , với 0;
2
t
π 
∈  
hoặc cosx y= , với 0;
2
y
π 
∈   
 Nếu : x, y là hai số thực thỏa:
2 2
1x y+ = , thì đặt sin , cosx t y t= = với 0 2t π≤ ≤
20
 Nếu x a≥ , ta có thể đặt :
sin
a
x
t
= , với ;
2 2
t
π π 
∈ − ÷
 
, tương tự cho trường hợp khác
 x là số thực bất kỳ thi đặt : tan , ;
2 2
x t t
π π 
= ∈ − ÷
 
Tại sao lại phải đặt điều kiện cho t như vậy ?
Chúng ta biết rằng khi đặt điều kiện ( )x f t= thì phải đảm bảo với mỗi x có duy nhất một t , và
điều kiện trên để đảm bào điều này . (xem lại vòng tròn lượng giác )
2. Xây dựng phương trình vô tỉ bằng phương pháp lượng giác như thế nào ?
Từ công phương trình lượng giác đơn giản: cos3 sint t= , ta có thể tạo ra được phương trình vô tỉ
Chú ý : 3
cos3 4cos 3cost t t= − ta có phương trình vô tỉ: 3 2
4 3 1x x x− = − (1)
Nếu thay x bằng
1
x
ta lại có phương trình : 2 2 2
4 3 1x x x− = − (2)
Nếu thay x trong phương trình (1) bởi : (x-1) ta sẽ có phương trình vố tỉ khó:
3 2 2
4 12 9 1 2x x x x x− + − = − (3)
Việc giải phương trình (2) và (3) không đơn giản chút nào ?
Tương tự như vậy từ công thức sin 3x, sin 4x,…….hãy xây dựng những phương trình vô tỉ theo
kiểu lượng giác .
3. Một số ví dụ
Bài 1. Giải phương trình sau : ( ) ( )
2
3 32 2 1
1 1 1 1
33
x
x x x
− + − + − − = +
  
Giải:
Điều kiện : 1x ≤
Với [ 1;0]x∈ − : thì ( ) ( )
3 3
1 1 0x x+ − − ≤ (ptvn)
[0;1]x∈ ta đặt : cos , 0;
2
x t t
π 
= ∈  
. Khi đó phương trình trở thành:
1 1
2 6 cos 1 sin 2 sin cos
2 6
x t t t
 
+ = + ⇔ = ÷
 
vậy phương trình có nghiệm :
1
6
x =
Bài 2. Giải các phương trình sau :
1)
1 2 1 2
1 2 1 2
1 2 1 2
x x
x x
x x
− +
− + + = +
+ −
HD:
1 2cos
tan
1 2cos
x
x
x
+
=
−
2) ( )2 2
1 1 1 2 1x x x+ − = + − Đs:
1
2
x =
3) 3
3 2x x x− = + HD: chứng minh 2x > vô nghiệm
Bài 3 . Giải phương trình sau: 3
6 1 2x x+ =
Giải: Lập phương 2 vế ta được:
3 3 1
8 6 1 4 3
2
x x x x− = ⇔ − =
Xét : 1x ≤ , đặt [ ]cos , 0;x t t π= ∈ . Khi đó ta được
5 7
cos ;cos ;cos
9 9 9
S
π π π 
=  
 
mà phương trình bậc 3
có tối đa 3 nghiệm vậy đó cũng chính là tập nghiệm của phương trình.
Bài 4. .Giải phương trình
2
2
1
1
1
x
x
 
+ ÷
− 
Giải: đk: 1x > , ta có thể đặt
1
, ;
sin 2 2
x t
t
π π 
= ∈ − ÷
 
21
Khi đó ptt: ( )2
cos 0
1
1 cot 1 1
sin sin2
2
t
t
x t
=
+ = ⇔
 = −

Phương trình có nghiệm : ( )2 3 1x = − +
Bài 5 .Giải phương trình :
( )
( )
222
2
2
11
1
2 2 1
xx
x
x x x
++
+ = +
−
Giải: đk 0, 1x x≠ ≠ ±
Ta có thể đặt : tan , ;
2 2
x t t
π π 
= ∈ − ÷
 
Khi đó pttt. ( )2
2sin cos2 cos2 1 0 sin 1 sin 2sin 0t t t t t t+ − = ⇔ − − =
Kết hợp với điều kiện ta có nghiệm
1
3
x =
Bài tập tổng hợp
Giải các phương trình sau
( )
33 2 2
1 2 2x x x x+ − = −
2
2 2 30 2007. 30 4 2007 30. 2007x x x− − + =
2
12 8
2 4 2 2
9 16
x
x x
x
−
+ − − >
+
33 3
1 1 2x x x− + + =
3 3
1 2 1x x x+ + = +
4 5 3 1 2 7 3x x x x+ + + = + + +
( )2 2
3 1 3 1x x x x+ + = + +
4 3 10 3 2x x− − = − (HSG Toàn Quốc 2002)
( ) ( ) ( ) ( )2 2 5 2 10x x x x x− − = + − −
23
4 1 2 3x x x+ = − + −
2 33
1 3 2 3 2x x x− + − = −
2 3
2 11 21 3 4 4 0x x x− + − − = (OLYMPIC 30/4-2007)
2 2 2 2
2 1 3 2 2 2 3 2x x x x x x x− + − − = + + + − +
2 2
2 16 18 1 2 4x x x x+ + + − = +
2
2 3 3 2
2
3 1
x x
x x
x
+ +
+ + =
+
12 2 1 3 9x x x+ − = +
3 244
1 1x x x x+ + = + +
2
4 3 3 4 3 2 2 1x x x x x+ + = + + −
3 2 4
1 1 1 1x x x x x− + + + + = + −
( ) ( ) ( )2 2
4 2 4 16 2 4 16 2 9 16x x x x+ + − + − = +
2
(2004 )(1 1 )x x x= + − −
( 3 2)( 9 18) 168x x x x x+ + + + =
2 4 23
3 1 1
3
x x x x− + = − + +
( ) ( )
2 2233 3
2 1 3 1 1 0x x x+ + − + − =
2
2008 4 3 2007 4 3x x x− + = −
( ) ( )2 2
3 2 1 1 1 3 8 2 1x x x x+ − = + + +
2
12 1 36x x x+ + + =
( ) 3 3
4 1 1 2 2 1x x x x− + = + +
1 1 1
2 1 3
x
x x
x x x
−
+ = − + −
2 2
5 14 9 20 5 1x x x x x− + − − − = +
33
6 1 8 4 1x x x+ = − −
( ) ( )215
30 4 2004 30060 1 1
2
x x x− = + +
24 9
7 7
28
x
x x
+
= +
2 2
4 4 10 8 6 10x x x x− − = − −
3 x x x x− = +
CHUYÊN ĐỀ: PHƯƠNG TRÌNH VÔ TỶ
22
I. PHƯƠNG PHÁP BIỂN ĐỔI TƯƠNG ĐƯƠNG
Dạng 1 : Phương trình
(*)
0
x D
A B A B
A B
∈
= ⇔ = ≥ ⇔ 
=
Lưu ý: Điều kiện (*) được chọn tuỳ thuôc vào độ phức tạp của 0A ≥ hay 0B ≥
Dạng 2: Phương trình 2
0B
A B
A B
≥
= ⇔ 
=
Dạng 3: Phương trình
+)
0
0
2
A
A B C B
A B AB C
 ≥

+ = ⇔ ≥

+ + =
(chuyển về dạng 2)
+) ( )3 3 3 33 3
3 .A B C A B A B A B C+ = ⇒ + + + =
và ta sử dụng phép thế : 3 3
A B C+ = ta được phương trình : 3
3 . .A B A B C C+ + =
Bài 1: Giải phương trình:
a) 2
1 1x x− = −
b) 2 3 0x x− + =
c) 2
1 1x x+ + =
e) 3 2 1 3x x− + − =
f) 3 2 1x x+ − − =
g) 9 5 2 4x x+ = − +
h) 3 4 2 1 3x x x+ − + = +
i) 2 2
( 3) 10 12x x x x+ − = − −
II.PHƯƠNG PHÁP ĐẶT ẨN PHỤ
Phương pháp đặt ẩn phụ thông thường.
-Nếu bài toán có chứa ( )f x và ( )f x khi đó đặt ( )t f x= (với điều kiện tối thiểu là 0t ≥ . đối với các
phương trình có chứa tham số thì nhất thiết phải tìm điều kiện đúng cho ẩn phụ).
-Nếu bài toán có chứa ( )f x , ( )g x và ( ). ( )f x g x k= (với k là hằng số) khi đó có thể đặt :
( )t f x= , khi đó ( )
k
g x
t
=
-Nếu bài toán có chứa ( ) ( ) ; ( ). ( )f x g x f x g x± và ( ) ( )f x g x k+ = khi đó có thể đặt:
( ) ( )t f x g x= ± suy ra
2
( ). ( )
2
t k
f x g x
−
=
-Nếu bài toán có chứa 2 2
a x− thì đặt sinx a t= với
2 2
t
π π
− ≤ ≤ hoặc cosx a t= với 0 t π≤ ≤
-Nếu bài toán có chứa 2 2
x a− thì đặt
sin
a
x
t
= với { };  0
2 2
t
π π 
∈ −  
hoặc
cos
a
x
t
= với
[ ]0; 
2
t
π
π
 
∈  
 
Bài 2: Tìm m để phương trình sau có nghiệm: 2 2
3 2 2x x m x x− + − = + −
Bài 3: Cho phương trình: 2
1x x m− − =
-Giải phương trình khi m=1
-Tìm m để phương trình có nghiệm.
Bài 4: Cho phương trình: 2
2 3x mx x m+ − = −
-Giải phương trình khi m=3
-Với giá trị nào của m thì phương trình có nghiệm.
23
-Nếu bài toán có chứa 2 2
x a+ ta có thể đặt .tanx a t= với ;
2 2
t
π π 
∈ − ÷
 
24
Bài 1: Giải phương trình:
a) 2 2
2 8 12 2x x x x+ + + = −
b) 2 2
2 5 2 3 9 3 3x x x x− + + = − −
c) 2 2
4 6 2 8 12x x x x− + = − +
d) 2 2
3 15 2 5 1 2x x x x+ + + + =
e) 2
( 4)( 1) 3 5 2 6x x x x+ + − + + =
f) 2 2
2 5 2 2 2 5 6 1x x x x+ + − + − =
g) 2 2
3 2 2 2 6 2 2x x x x+ + − + + = −
h) 2 2
11 31x x+ + =
i) 2
( 5)(2 ) 3 3x x x x+ − = +
Bài 2: Giải phương trình:
a) ( ) ( )
33 2 2
1 2 1x x x x+ − = −
b)
( ) ( )
3 32 2
1 1 1 1 2 1x x x x + − − − + = + −
  
c) 2 2
1 2 1 2 1 0x x x x− − − − + =
d) 6 4 2 2
64 112 56 7 2 1x x x x− + − = −
e) 2
35
121
x
x
x
+ =
−
f) ( ) ( ) ( )
1
3 1 4 3 3
3
x
x x x
x
+
− + + − = −
−
Bài 4: Cho phương trình: 2
1 1
1
m
x x
+ =
−
-Giải phương trình với
2
2
3
m = +
-Tìm m để phương trình có nghiệm.
Bài 5: Cho phương trình: ( )2 2
2 2 2 3 0x x x x m− + − − − =
-Giải phương trình với m = 9
-Tìm m để phương trình có nghiệm.
2. Phương pháp đặt ẩn phụ không hoàn toàn
Là việc sử dụng một ẩn phụ chuyển phương trình ban đầu thành một phương trình với một ẩn phụ nhưng các hệ
số vẫn còn chứa x.
-Từ những phương trình tích ( )( )1 1 1 2 0x x x+ − + − + = ,( )( )2 3 2 3 2 0x x x x+ − + − + =
Khai triển và rút gọn ta sẽ được những phương trình vô tỉ không tầm thường chút nào, độ khó của phương trình
dạng này phụ thuộc vào phương trình tích mà ta xuất phát.
Từ đó chúng ta mới đi tìm cách giải phương trình dạng này .Phương pháp giải được thể hiện qua các ví dụ sau .
Bài 1. Giải phương trình : ( )2 2 2
3 2 1 2 2x x x x+ − + = + +
Giải: 2
2t x= + , ta có : ( )2 3
2 3 3 0
1
t
t x t x
t x
=
− + − + = ⇔  = −
25
Bài 2. Giải phương trình : ( ) 2 2
1 2 3 1x x x x+ − + = +
Giải:
Đặt : 2
2 3, 2t x x t= − + ≥
Khi đó phương trình trở thnh : ( ) 2
1 1x t x+ = + ( )2
1 1 0x x t⇔ + − + =
Bây giờ ta thêm bớt , để được phương trình bậc 2 theo t có ∆ chẵn
( ) ( ) ( ) ( )2 2 2
2 3 1 2 1 0 1 2 1 0
1
t
x x x t x t x t x
t x
=
− + − + + − = ⇔ − + + − = ⇔  = −
Từ một phương trình đơn giản : ( )( )1 2 1 1 2 1 0x x x x− − + − − + + = , khai triển ra ta sẽ được pt sau
Bài 3. Giải phương trình sau : 2
4 1 1 3 2 1 1x x x x+ − = + − + −
Giải:
Nhận xét : đặt 1t x= − , pttt: 4 1 3 2 1x x t t x+ = + + + (1)
Ta rt 2
1x t= − thay vo thì được pt: ( ) ( )2
3 2 1 4 1 1 0t x t x− + + + + − =
Nhưng không có sự may mắn để giải được phương trình theo t ( ) ( )
2
2 1 48 1 1x x∆ = + + − + − không có
dạng bình phương .
Muốn đạt được mục đích trên thì ta phải tách 3x theo ( ) ( )
2 2
1 , 1x x− +
Cụ thể như sau : ( ) ( )3 1 2 1x x x= − − + + thay vào pt (1) ta được:
Bài 4. Giải phương trình: 2
2 2 4 4 2 9 16x x x+ + − = +
Giải .
Bình phương 2 vế phương trình: ( ) ( ) ( )2 2
4 2 4 16 2 4 16 2 9 16x x x x+ + − + − = +
Ta đặt : ( )2
2 4 0t x= − ≥ . Ta được: 2
9 16 32 8 0x t x− − + =
Ta phải tách ( ) ( )2 2 2
9 2 4 9 2 8x x xα α α= − + + − làm sao cho t∆ có dạng chình phương .
Nhận xét : Thông thường ta chỉ cần nhóm sao cho hết hệ số tự do thì sẽ đạt được mục đích.
Bài tập: Giải các phương trình sau:
a) 3 3
(4 1) 1 2 2 1x x x x− + = + + b) 2 2
1 2 2x x x x− = −
c) 2 2
1 2 2x x x x− = + d) 2 2
4 ( 2) 2 4x x x x x+ = + − +
3. Phương pháp đặt ẩn phụ chuyển về hệ.
a) Dạng thông thường: Đặt ( ) ( ),u x v xα β= = và tìm mối quan hệ giữa ( )xα và ( )xβ từ đó tìm được hệ
theo u,v. Chẳng hạn đối với phương trình: ( ) ( )m ma f x b f x c− + + = ta có thể đặt:
( )
( )
m
m
u a f x
v b f x
 = −

= +
từ đó
suy ra m m
u v a b+ = + . Khi đó ta có hệ
m m
u v a b
u v c
 + = +

+ =
Bài tập: Giải các phương trình sau:
a) 3
2 1 1x x− = − − b) 3
9 2 1x x− = − − c) 2
1 ( 1) 0x x x x x x− − − − + − =
b) Dạng phương trình chứa căn bậc hai và lũy thừa bậc hai:
26
2
( )ax b c dx e xα β+ = + + + với
d ac
e bc
α
β
= +

= +
Cách giải: Đặt: dy e ax b+ = + khi đó phương trình được chuyển thành hệ:
( )
( )
2
22
( )
dy e ax bdy e ax b
dy e c dx e x c dy e x dy eα β α β
 + = + + = + 
⇔ 
+ = + + + + = − + + − 
->giải
Nhận xét: Dể sử dụng được phương pháp trên cần phải khéo léo biến đổi phương trình ban đầu về dạng thỏa
mãn điều kiện trên để đặt ẩn phụ.Việc chọn ;α β thông thường chúng ta chỉ cần viết dưới dạng :
( ) ' '
n n
x p a x bα β γ+ = + + là chọn được.
c) Dạng phương trình chứa căn bậc ba và lũy thừa bậc ba.
( )
33
ax b c dx e xα β+ = + + + với
d ac
e bc
α
β
= +

= +
Cách giải: Đặt 3
dy e ax b+ = + khi đó phương trình được chuyển thành hệ:
( )
( )
( )
( )
3 33
3 3 3
( ) ( )
dy e ax bdy e ax b c dy e acx bc
dy e c dx e x c dx e ac d x dy bcc dx e x dy e
 + = ++ = + + = +  
⇔ ⇔  
+ = + + + + = − + ++ = − + + −    α β α β
Bài tập: Giải các phương trình sau:
1) 2
1 4 5x x x+ = + +
2) 2
3 1 4 13 5x x x+ = − + −
3) 3 3
2 3 3 2x x+ = −
4) 24 9
7 7 0
28
x
x x x
+
= + >
5) 3 3
1 2 2 1x x+ = −
6) ( )3 33 3
35 35 30x x x x− + − =
7) 2
4 13 5 3 1 0x x x− + + + =
8) 2
4 13 5 3 1 0x x x− + + + =
( ) ( )215
30 4 2004 30060 1 1
2
x x x− = + +
3 23
3 5 8 36 53 25x x x− = − + −
9)
3 23 4
81 8 2 2
3
x x x x− = − + −
10) 33
6 1 8 4 1x x x+ = − −
II. PHƯƠNG PHÁP HÀM SỐ
Sử dụng các tính chất của hàm số để giải phương trình là dạng toán khá quen thuộc. Ta có 3 hướng áp dụng sau
đây:
Hướng 1: Thực hiện theo các bước:
Bước 1: Chuyển phương trình về dạng: ( )f x k=
Bước 2: Xét hàm số ( )y f x=
Bước 3: Nhận xét:
• Với 0 0( ) ( )x x f x f x k= ⇔ = = do đó 0x là nghiệm
• Với 0 0( ) ( )x x f x f x k> ⇔ > = do đó phương trình vô nghiệm
• Với 0 0( ) ( )x x f x f x k< ⇔ < = do đó phương trình vô nghiệm
• Vậy 0x là nghiệm duy nhất của phương trình
Hướng 2: thực hiện theo các bước
Bước 1: Chuyển phương trình về dạng: ( ) ( )f x g x=
Bước 2: Dùng lập luận khẳng định rằng ( )f x và g(x) có những tính chất trái ngược nhau và xác định 0x sao
cho 0 0( ) ( )f x g x=
27
Bước 3: Vậy 0x là nghiệm duy nhất của phương trình.
Hướng 3: Thực hiện theo các bước:
Bước 1: Chuyển phương trình về dạng ( ) ( )f u f v=
Bước 2: Xét hàm số ( )y f x= , dùng lập luận khẳng định hàm số đơn điệu
Bước 3: Khi đó ( ) ( )f u f v u v= ⇔ =
Ví dụ: Giải phương trình : ( ) ( ) ( )2 2
2 1 2 4 4 4 3 2 9 3 0x x x x x+ + + + + + + =
pt ( ) ( )( ) ( ) ( )( ) ( ) ( )
2 2
2 1 2 2 1 3 3 2 3 3 2 1 3x x x x f x f x⇔ + + + + = − + − + ⇔ + = −
Xét hàm số ( ) ( )2
2 3f t t t= + + , là hàm đồng biến trên R, ta có
1
5
x = −
Bài tập: Giải phương trình:
2
4 1 4 1 1x x− + − = , 3
1 4 5x x x− = − − + , 2
1 3x x x− = + − , 2 3
1 2 2x x x x= − + − ,
1 2 3x x− + + = , 2
2 1 3 4x x x− + + = −
28
BAØI TAÄP :
Baøi 1: Bình phöông hai veá :
a) x
2
+ 1 1x + =
Hd: pt 4 2
0
1 1
1
2 0
1 5
2
x
x
x
x x x
x

 =
− ≤ ≤
⇔ =−
− − = 
± =

b)pt:
5 1 3 2 1 0
: 1
x x x
dk x
− − − − − =
≥
- Chuyeån veá ,bình phöông hai veá : x =2 ;
x = 2/11( loaïi ) . Vaäy x=2 .
c)
: 9 5 2 4
: 2
pt x x
dk x
+ = − +
≥
Bình phöông hai laà ta coù :ÑS x = 0 .
d)
: 16 9 7
: 0; 7
pt x x
Ds x
− + + =
= −
e)
2 2
:(4 1) 9 2 2 1
: 1/ 4
pt x x x x
dk x
− + = + +
≥
Bphöông hai lanà ta coù :ÑS x = 4/3
Baøi 2 : Daët Aån soá phuï :
a)
2 2
3 3 3 6 3x x x x− + + − + =
- Ñaët :
- T=x
2
-3x+3
3 / 4 : 3 3
1 1; 2
pt t t
t x
≥ + + =
<=> = => =
b)
22
1 1 0
3
: 0 1
x x x x
dk x
+ − = + − =
≤ ≤
- Ñaët :
2
2 1
1 ; 0
2
t
t x x t x x
−
= + − ≥ => − =
ptt
2
-3t +2 =0 t =1 ; t=2 Vn
t=1  x=0 ; x=1 .
c)
2
2 3 1 3 2 2 5 3 16x x x x x+ + + = + + + −
HDÑS:
29
ÑK : 2 2
1
2 3 1 0
3 4 2 2 5 3
5 3.
x
t x x
t x x x
pt t x
≥ −
= + + + ≥
=> = + + + +
<=> = <=> =
2 2 2
2
) 7 2 3 3 19
. 2 7 / 4
5 3 13 4
1; 2
d x x x x x x
t x x
pt t t t t
x x
+ + + + + = + +
= + + ≥
<=> + + = + <=> =
=> = = −
3: ) 1 3 ( 1)(3 )Bai a x x x x m+ + − − + − =
• Giaûi pt khi m=2 .** Tìm m pt coù nghieäm .
• HDÑS : ÑK:
2
. 1 3 ; 2 2 2
: 2( )
0( )
) 2: 2 0 1, 3
2
t x x t
vi a b a b a b
t l
a m t t x x
t
= + + − => ≤ ≤
+ ≤ + ≤ +
=
= − = <=> => = − = =
b) f(t) = -t
2
/2 + t +2 = m (1) . Laäp baûng bieán
thieân : Tacoù : 2 2 2 2.m− ≤ ≤
2
4: ) 9 9bai a x x x x m+ − = − + +
Bình phöông : Ñaët t= (9 ) 0 9/ 2x x t− => ≤ ≤
KsHS
2
( ) 2 9 ; 9/ 2 9/ 4 10f t t t o t Ds m= − + + ≤ ≤ − ≤ ≤ d)
4 44
4 4 6x x m x x m+ + + + + =
HDÑS:Ñaët :
4 24
44
4
4 0 : 6 0
3
2
4 2
4 16
t x x m pt t t
t l
t
x x m
m x x
= + + ≥ + − =
=−
<=> =
=> + + =
<=> =− − +
Laäp BBT : m>19VN; m=19: 1 ngh ;m<19pt2ngh.
Baøi3:
1-
BAØI TAÄP : I- GIAÛI PT:
3 3
1) 2 2 3 1X X− + − = Laäp phöông hai veá ta coù : x
3
-4x
2
+5x-2 =0 x=1 ; x=2 . Thöû laïi x=1 khoâng
thoaû .Vaäy x=2 .
30
3 3
2) 34 3 1X X+ − − = Laäp phöông hai veá ta coù : x
2
+31x-1830 =0 x=-1061 ; x=75 .
3 3
2) 2 2 3 1X X+ + − =
-Laäp phöông hai veá ta coù : x
3
-4x
2
+5x-2 =0 x=1 ; x=2 . Thöû laïi x=1 khoâng thoaû .Vaäy x=2 .
3 3 3
3) 1 2 2 3X X x− + − = −
-Laäp phöông hai veá ta coù : pt x=1 ; x=2 ;x=3/2. Thöû laïi Ñeàu thoaû .
3 3 3
4) 2 2 2 9X X x+ + − =
--Laäp phöông hai veá ta coù : pt x=0 ; x=3 ;x=-6/5. Thöû laïi Ñeàu thoaû .
32 2 2 23 3
5) ( ) 2 ( ) 0X a X a x a a+ + + − = − ≠ pt 18X = 14a  x=7a/ 9 ; a# 0 . Thöû laïi thoaû.
II- PHÖÔNG TRÌNH CAÊN THÖÙC CHÖÙA THAM SOÁ m :
Baøi 1: 2 1x x m+ = + : BLs ngh pt.
HdÑS :
[ )1;
: ( ) 2 1
D
xet f x x x
= − +∞
− = + −
Tính ñaïo haøm : Baûng bieán thieân ,Ta coù :
m<1 : 1ngh; m=1: coù 2ngh: 1<m<2: 2ngh
m=2 : 2ngh ; 2<m< +∞ : 0 Vn .
BAÁT PHÖÔNG TRÌNH CAÊN THÖÙC
KIEÁN THÖÙC CAÀN NHÔÙ :
• Daïng cô baûn :
2
2
2
2
0
0
0
0
0
0
0
0
0
0
A
A B B
A B
A
A B B
A B
A
B
A B
B
A B
A
B
A B
B
A B
 ≥

< ⇔ >
 <
 ≥

≤ ⇔ ≥
 ≤
 ≥

<> ⇔ ≥

>
 ≥

≤≥ ⇔ >

≥
• Daïng khaùc :
- Coù nhieàu caên thöùc :Ñaët ÑK – Luyõ thöøa- khöû caên – Döa veå bpt cô baûn nhö caùc daïng
treân .
Chuù yù : - Hai veá khoâng aâm ta ñ7ôïc bình phöông – Hai veá laø soá thöïc ta ñöïôc laäp phöông .
31
BAØI TAÄP :
GIAÛI CAÙC BAÁT PHÖÔNG TRÌNH
1-Pt : 2 3 2x x+ ≥ −
pt -3/2 3 2 2x≤ ≤ +
32
PHÖÔNG TRÌNH CAÊN THÖÙC
III. PHƯƠNG PHÁP BIỂN ĐỔI TƯƠNG ĐƯƠNG
Dạng 1 : Phương trình
(*)
0
x D
A B A B
A B
∈
= ⇔ = ≥ ⇔ 
=
Lưu ý: Điều kiện (*) được chọn tuỳ thuôc vào độ phức tạp của 0A ≥ hay 0B ≥
Dạng 2: Phương trình 2
0B
A B
A B
≥
= ⇔ 
=
Dạng 3: Phương trình
+)
0
0
2
A
A B C B
A B AB C
 ≥

+ = ⇔ ≥

+ + =
(chuyển về dạng 2)
+) ( )3 3 3 33 3
3 .A B C A B A B A B C+ = ⇒ + + + =
và ta sử dụng phép thế : 3 3
A B C+ = ta được phương trình : 3
3 . .A B A B C C+ + =
Bài tập trong các đề thi tuyển sinh.
Bài 1 :
a)(ĐHXD) Giải pt 2
6 6 2 1x x x− + = −
b) (CĐSP MG 2004) 2
4 3 2 5x x x− + − = −
c) (CĐSP NINH BÌNH) 3 2 7 1x x− − + =
d) (CĐ hoá chất) 8 3x x x+ − = +
e) (CĐ TP 2004) 2 2 1 7x x− − =
g) (CĐSP bến tre) 5 1 3 2 1 0x x x− − − − − =
h) (CĐ truyền hình 2007) 2 2
7 5 3 2x x x x x− + + = − −
ĐS:
a) x=1. b) x=14/5 c) x=9. d)x=1
e) x=5 g) x=2 h) x=-1.
Bài 2:
a)(ĐHNN-2001) Giải phương trình 1 4 ( 1)(4 ) 5.x x x x+ + − + + − =
b) (CĐ Nha trang 2002) : 2 5 ( 2)(5 ) 4x x x x+ + − + + − =
Hdẫn:
a) ĐK: -1≤x≤4.
Đặt t= 1 4 0x x+ + − > . Giải được t=-5 (loại), t=3. Giải t=3 được x=0.
b) x=
3 3 5
2
±
Bài 3
a)(ĐHQG KD-2001) Giải phương trình 2
4 1 4 1 1x x− + − = .
b) (CĐXD 2003) 3 3 3
2 1 2 2 2 3 0x x x+ + + + + =
33
Hdẫn:
a) ĐK: x≥1/2
Xét hàm số y= 2
4 1 4 1x x− + − . HSĐB trên [1/2;+∞). Và f(1/2)=1.
Vậy phương trình có nghiệm duy nhất x=1/2.
b)x=-1 là nghiệm .
Các hàm số y= 3
2 1x + ; y= 3
2 2x + ; y= 3
2 3x + ĐB
Bài 4 : Giải pt 2 2
2 8 6 1 2 2x x x x+ + + − = + .
ĐK : x ≤-3,x=-1,x≥1.
-Với x=-1 Thoả mãn pt
-Với x≤-3 thì VP<0 loại
-Với x≥1 pt
2
( 1)(2 6) ( 1)( 1) 2 ( 1)
2 6 1 2 1
x x x x x
x x x
⇔ + + + + − = +
⇔ + + − = +
Tiếp tục bình phương 2 vế thu được x=1.
Vậy pt có 2 nghiệm x=1 ; x=-1.
Bài 5 : (ĐH mỏ điạ chất) Giải pt 2 2
4 2 3 4x x x x+ − = + −
ĐK : 2x ≤ . Đặt t= 2
4x x+ − . Giải được t=2 ; t=-4/3.
+t=2 được x=0, x=2
+t=-4/3 được
2 14 2 14
;
3 3
x x
− − − +
= = (loại)
KL : Pt có 3 nghiệm.
Bài 6 : (HV CNBCVT) Giải pt
3
4 1 3 2
5
x
x x
+
+ − − = .
Giải : ĐK : x≥2/3.
Trục căn thức ta được
3
3 ( 4 1 3 2) 4 1 3 2 5
5
x
x x x x x
+
+ = + + − ⇔ + + − = .
PT trên có nghiệm x=2.
HS y= 4 1 3 2x x+ + − ĐB do vậy x=2 là nghiệm duy nhất.
Bài 7: Giải phương trình 3(2 2) 2 6x x x+ − = + + .
ĐK: x≥2.
pt
2(3 ) 6 2 2
2(3 )( 6 2 2) 8(3 )
3
6 2 2 4
x x x
x x x x
x
x x
⇔ − = + − −
⇔ − + + − = −
=
⇔ 
+ + − =
KL: x=3; x=
11 15
2
−
34
Bài 8: Giải phương trình 2
7 7x x+ + =
ĐK:x≥-7.
Đặt 2
7 0 7t x t x= + ≥ ⇒ = + .
Phương trình trở thành
2
2 2
2
( ) ( )( 1) 0
7
x t
x t x t x t x t
t x
 + =
⇒ − = − + ⇔ + − + =
= +
Giải được x=2; x=
1 29
2
−
a) + = −3 3
1 2 2 1x x
+ = −
= − ⇔ + =
3 3
33
1 2 2 1
2 1 1 2
x x
y x y x
- Ph¬ng tr×nh ®îc chuyÓn thµnh hÖ

 =  = =
 
+ = + = + =  − +   ⇔ ⇔ ⇔ = =   + = − = − − + + + =      − − + = = =
33 3
3 3 3 2 2
3
1
1 21 2 1 2 1 5
21 2 2( ) 2 0( )
1 51 2
2
x y x y
x yx y x y
x y
y x x y x y x xy y vn
x y x y
- VËy ph¬ng tr×nh ®· cho cã 3 nghiÖm.
c)
2 23 3 3(2 ) (7 ) (7 )(2 ) 3x x x x− + + − − − =
-Ñaët :
2 23 32
.
3 33 7 9
3
1; 2 1; 6
2
u v uvu x
pt
v x u v
u v
u v x
uv

 
 
  



+ − == −
<=>
= + + =
+ =
<=> <=> = = => = −
=
d)
3
2 1 1x x− = − −
.ÑK : x 1≥
3 2
1; 0
1
0;1; 2; 1;0;33 2 1
1;2;10
u x
v x v
u v
u v
u v
x







= −
= − ≥
= −
=> <=> = − =
+ =
=
Bài 9: Giải phương trình 2
2 2 2 4 2 2x x x x− − + = − − +
ĐK: x≥2.
Đặt 2 2
2 2 2 4 2t x x t x x= − − + ⇒ = − − + .
Thế vào phương trình giải được t=1; t=-2. từ đó giải được x=2.
Bài 10: (Tham khảo 2002) giải phương trình 2
4 4 2 12 2 16x x x x+ + − = − + −
35
ĐK:x≥4.
Phương trình 2
4 4 ( 4 4) 12x x x x⇔ + + − = + + − −
Đặt t= 4 4x x+ + − ≥0. giải phương trình ẩn t được t=4; t=-3 (loại).
Giải được x=5.
Bài 11 :
a)(CĐSP 2004) Giải pt
3
2 1 2 1
2
x
x x x x
+
+ − + − − =
b) (ĐH-KD-2005) 2 2 2 1 1 4x x x+ + + − + =
a) ĐK ; x≥1.
Pt
3
1 1 1 1
2
x
x x
+
⇔ − + + − − = .
Xét 1≤x≤2 : giải được nghiệm x=1
xét x>2 giải được x=5
b)x=3
3. Phương pháp đặt ẩn phụ không hoàn toàn
 Từ những phương trình tích ( )( )1 1 1 2 0x x x+ − + − + = ,( )( )2 3 2 3 2 0x x x x+ − + − + =
Khai triển và rút gọn ta sẽ được những phương trình vô tỉ không tầm thường chút nào, độ khó
của phương trình dạng này phụ thuộc vào phương trình tích mà ta xuất phát .
Từ đó chúng ta mới đi tìm cách giải phương trình dạng này .Phương pháp giải được thể hiện
qua các ví dụ sau .
Bài 1. Giải phương trình : ( )2 2 2
3 2 1 2 2x x x x+ − + = + +
Giải:
2
2t x= + , ta có : ( )2 3
2 3 3 0
1
t
t x t x
t x
=
− + − + = ⇔  = −
Bài 2. Giải phương trình : ( ) 2 2
1 2 3 1x x x x+ − + = +
Giải:
Đặt : 2
2 3, 2t x x t= − + ≥ Khi đó phương trình trở thnh : ( ) 2
1 1x t x+ = +
( )2
1 1 0x x t⇔ + − + =
Bây giờ ta thêm bớt , để được phương trình bậc 2 theo t có ∆ chẵn :
( ) ( ) ( ) ( )2 2 2
2 3 1 2 1 0 1 2 1 0
1
t
x x x t x t x t x
t x
=
− + − + + − = ⇔ − + + − = ⇔  = −
36

Weitere ähnliche Inhalte

Was ist angesagt?

Tuyen tap cac bai toan va phuong phap giai pt va bpt vo ty
Tuyen tap cac bai toan va phuong phap giai  pt va bpt vo ty Tuyen tap cac bai toan va phuong phap giai  pt va bpt vo ty
Tuyen tap cac bai toan va phuong phap giai pt va bpt vo ty Huynh ICT
 
[Vnmath.com] phuong-trinh-bpt-trong-de-thi-thu-2014
[Vnmath.com] phuong-trinh-bpt-trong-de-thi-thu-2014[Vnmath.com] phuong-trinh-bpt-trong-de-thi-thu-2014
[Vnmath.com] phuong-trinh-bpt-trong-de-thi-thu-2014Antonio Krista
 
Chuyen de phuong trinh he phuong trinh on thi dai hoc
Chuyen de phuong trinh he phuong trinh on thi dai hocChuyen de phuong trinh he phuong trinh on thi dai hoc
Chuyen de phuong trinh he phuong trinh on thi dai hocVui Lên Bạn Nhé
 
Boxmathtuyentaphept.thuvienvatly.com.57d61.18410
Boxmathtuyentaphept.thuvienvatly.com.57d61.18410Boxmathtuyentaphept.thuvienvatly.com.57d61.18410
Boxmathtuyentaphept.thuvienvatly.com.57d61.18410Cuong Archuleta
 
Hệ phương trình
Hệ phương trìnhHệ phương trình
Hệ phương trìnhtuituhoc
 
Kỹ thuật nhân liên hợp
Kỹ thuật nhân liên hợpKỹ thuật nhân liên hợp
Kỹ thuật nhân liên hợptuituhoc
 
7 3-2016 tuyen tap 50 bai he pt hay va dac sac tang hs online-p1
7 3-2016 tuyen tap 50 bai he pt hay va dac sac tang hs online-p17 3-2016 tuyen tap 50 bai he pt hay va dac sac tang hs online-p1
7 3-2016 tuyen tap 50 bai he pt hay va dac sac tang hs online-p1Nguyen Tan
 
Các phương pháp hay giải Phuong trinh-vo-ty
Các phương pháp hay giải Phuong trinh-vo-tyCác phương pháp hay giải Phuong trinh-vo-ty
Các phương pháp hay giải Phuong trinh-vo-tyroggerbob
 
Chuyên đề hệ phương trình bằng phương pháp hàm số
Chuyên đề hệ phương trình bằng phương pháp hàm sốChuyên đề hệ phương trình bằng phương pháp hàm số
Chuyên đề hệ phương trình bằng phương pháp hàm sốVui Lên Bạn Nhé
 
1.3 bien luan_pt_bang_do_thi
1.3 bien luan_pt_bang_do_thi1.3 bien luan_pt_bang_do_thi
1.3 bien luan_pt_bang_do_thivanthuan1982
 
Chuyen de he pt
Chuyen de he ptChuyen de he pt
Chuyen de he ptTam Ho Hai
 
Bat phuong trinh vo ti
Bat phuong trinh vo tiBat phuong trinh vo ti
Bat phuong trinh vo tiphongmathbmt
 
[Vnmath.com] bai giang-trong_tam_ve_ham_so_thay_dang_viet_hung
[Vnmath.com] bai giang-trong_tam_ve_ham_so_thay_dang_viet_hung[Vnmath.com] bai giang-trong_tam_ve_ham_so_thay_dang_viet_hung
[Vnmath.com] bai giang-trong_tam_ve_ham_so_thay_dang_viet_hungĐức Mạnh Ngô
 
04 phuong trinh mu p3
04 phuong trinh mu p304 phuong trinh mu p3
04 phuong trinh mu p3Huynh ICT
 
Bt toi uu hoa
Bt toi uu hoaBt toi uu hoa
Bt toi uu hoaThien Le
 
Tuyển tập 100 hệ phương trình thường gặp (2015-2016) - Megabook.vn
Tuyển tập 100 hệ phương trình thường gặp (2015-2016) - Megabook.vnTuyển tập 100 hệ phương trình thường gặp (2015-2016) - Megabook.vn
Tuyển tập 100 hệ phương trình thường gặp (2015-2016) - Megabook.vnMegabook
 
Ky thuat giai he phuong trinh
Ky thuat giai he phuong trinhKy thuat giai he phuong trinh
Ky thuat giai he phuong trinhHuynh ICT
 
04 phuong trinh mu p2
04 phuong trinh mu p204 phuong trinh mu p2
04 phuong trinh mu p2Huynh ICT
 

Was ist angesagt? (18)

Tuyen tap cac bai toan va phuong phap giai pt va bpt vo ty
Tuyen tap cac bai toan va phuong phap giai  pt va bpt vo ty Tuyen tap cac bai toan va phuong phap giai  pt va bpt vo ty
Tuyen tap cac bai toan va phuong phap giai pt va bpt vo ty
 
[Vnmath.com] phuong-trinh-bpt-trong-de-thi-thu-2014
[Vnmath.com] phuong-trinh-bpt-trong-de-thi-thu-2014[Vnmath.com] phuong-trinh-bpt-trong-de-thi-thu-2014
[Vnmath.com] phuong-trinh-bpt-trong-de-thi-thu-2014
 
Chuyen de phuong trinh he phuong trinh on thi dai hoc
Chuyen de phuong trinh he phuong trinh on thi dai hocChuyen de phuong trinh he phuong trinh on thi dai hoc
Chuyen de phuong trinh he phuong trinh on thi dai hoc
 
Boxmathtuyentaphept.thuvienvatly.com.57d61.18410
Boxmathtuyentaphept.thuvienvatly.com.57d61.18410Boxmathtuyentaphept.thuvienvatly.com.57d61.18410
Boxmathtuyentaphept.thuvienvatly.com.57d61.18410
 
Hệ phương trình
Hệ phương trìnhHệ phương trình
Hệ phương trình
 
Kỹ thuật nhân liên hợp
Kỹ thuật nhân liên hợpKỹ thuật nhân liên hợp
Kỹ thuật nhân liên hợp
 
7 3-2016 tuyen tap 50 bai he pt hay va dac sac tang hs online-p1
7 3-2016 tuyen tap 50 bai he pt hay va dac sac tang hs online-p17 3-2016 tuyen tap 50 bai he pt hay va dac sac tang hs online-p1
7 3-2016 tuyen tap 50 bai he pt hay va dac sac tang hs online-p1
 
Các phương pháp hay giải Phuong trinh-vo-ty
Các phương pháp hay giải Phuong trinh-vo-tyCác phương pháp hay giải Phuong trinh-vo-ty
Các phương pháp hay giải Phuong trinh-vo-ty
 
Chuyên đề hệ phương trình bằng phương pháp hàm số
Chuyên đề hệ phương trình bằng phương pháp hàm sốChuyên đề hệ phương trình bằng phương pháp hàm số
Chuyên đề hệ phương trình bằng phương pháp hàm số
 
1.3 bien luan_pt_bang_do_thi
1.3 bien luan_pt_bang_do_thi1.3 bien luan_pt_bang_do_thi
1.3 bien luan_pt_bang_do_thi
 
Chuyen de he pt
Chuyen de he ptChuyen de he pt
Chuyen de he pt
 
Bat phuong trinh vo ti
Bat phuong trinh vo tiBat phuong trinh vo ti
Bat phuong trinh vo ti
 
[Vnmath.com] bai giang-trong_tam_ve_ham_so_thay_dang_viet_hung
[Vnmath.com] bai giang-trong_tam_ve_ham_so_thay_dang_viet_hung[Vnmath.com] bai giang-trong_tam_ve_ham_so_thay_dang_viet_hung
[Vnmath.com] bai giang-trong_tam_ve_ham_so_thay_dang_viet_hung
 
04 phuong trinh mu p3
04 phuong trinh mu p304 phuong trinh mu p3
04 phuong trinh mu p3
 
Bt toi uu hoa
Bt toi uu hoaBt toi uu hoa
Bt toi uu hoa
 
Tuyển tập 100 hệ phương trình thường gặp (2015-2016) - Megabook.vn
Tuyển tập 100 hệ phương trình thường gặp (2015-2016) - Megabook.vnTuyển tập 100 hệ phương trình thường gặp (2015-2016) - Megabook.vn
Tuyển tập 100 hệ phương trình thường gặp (2015-2016) - Megabook.vn
 
Ky thuat giai he phuong trinh
Ky thuat giai he phuong trinhKy thuat giai he phuong trinh
Ky thuat giai he phuong trinh
 
04 phuong trinh mu p2
04 phuong trinh mu p204 phuong trinh mu p2
04 phuong trinh mu p2
 

Andere mochten auch

Toan daisotohop-chuong5(1)
Toan daisotohop-chuong5(1)Toan daisotohop-chuong5(1)
Toan daisotohop-chuong5(1)Huynh ICT
 
Xs cao hochsn
Xs cao hochsnXs cao hochsn
Xs cao hochsnHuynh ICT
 
Luong giac chuong 8
Luong giac chuong 8Luong giac chuong 8
Luong giac chuong 8Huynh ICT
 
So phuc va cac bai toan lien quan 2
So phuc va cac bai toan lien quan 2So phuc va cac bai toan lien quan 2
So phuc va cac bai toan lien quan 2Huynh ICT
 
10 dangtichphanhaygap thanhtung
10 dangtichphanhaygap thanhtung10 dangtichphanhaygap thanhtung
10 dangtichphanhaygap thanhtungHuynh ICT
 
Viet pt-mat-phang-nt long - www.mathvn.com
Viet pt-mat-phang-nt long - www.mathvn.comViet pt-mat-phang-nt long - www.mathvn.com
Viet pt-mat-phang-nt long - www.mathvn.comHuynh ICT
 
Tuyen tap cac bai toan hinh hoc giai tich trong khong gian (luyen thi dai hoc)
Tuyen tap cac bai toan hinh hoc giai tich trong khong gian (luyen thi dai hoc)Tuyen tap cac bai toan hinh hoc giai tich trong khong gian (luyen thi dai hoc)
Tuyen tap cac bai toan hinh hoc giai tich trong khong gian (luyen thi dai hoc)Huynh ICT
 
[Vnmath.com] pt-bpt-hpt-mu-logarit-dh-2002-2011
[Vnmath.com] pt-bpt-hpt-mu-logarit-dh-2002-2011[Vnmath.com] pt-bpt-hpt-mu-logarit-dh-2002-2011
[Vnmath.com] pt-bpt-hpt-mu-logarit-dh-2002-2011Huynh ICT
 
ham-so-on-thi-dh-huynh-bao-toan
 ham-so-on-thi-dh-huynh-bao-toan ham-so-on-thi-dh-huynh-bao-toan
ham-so-on-thi-dh-huynh-bao-toanHuynh ICT
 
Phan loai cac dang toan dai so to hop
Phan loai cac dang toan dai so to hopPhan loai cac dang toan dai so to hop
Phan loai cac dang toan dai so to hopHuynh ICT
 
Toan daisotohop-chuong5(2)
Toan daisotohop-chuong5(2)Toan daisotohop-chuong5(2)
Toan daisotohop-chuong5(2)Huynh ICT
 
Toan d dh_2011
Toan d dh_2011Toan d dh_2011
Toan d dh_2011Huynh ICT
 
Luonggiac chuong5
Luonggiac chuong5Luonggiac chuong5
Luonggiac chuong5Huynh ICT
 
Tich phan phamkimchung-www.mathvn.com
Tich phan phamkimchung-www.mathvn.comTich phan phamkimchung-www.mathvn.com
Tich phan phamkimchung-www.mathvn.comHuynh ICT
 
[Vnmath.com] 40 bai ham so chon loc nam 2013
[Vnmath.com] 40 bai ham so chon loc nam 2013[Vnmath.com] 40 bai ham so chon loc nam 2013
[Vnmath.com] 40 bai ham so chon loc nam 2013Huynh ICT
 
Luonggiac chuong2
Luonggiac chuong2Luonggiac chuong2
Luonggiac chuong2Huynh ICT
 
4 khao sat-ham_so_www.mathvn.com
4 khao sat-ham_so_www.mathvn.com4 khao sat-ham_so_www.mathvn.com
4 khao sat-ham_so_www.mathvn.comHuynh ICT
 

Andere mochten auch (18)

Mu va loga
Mu va logaMu va loga
Mu va loga
 
Toan daisotohop-chuong5(1)
Toan daisotohop-chuong5(1)Toan daisotohop-chuong5(1)
Toan daisotohop-chuong5(1)
 
Xs cao hochsn
Xs cao hochsnXs cao hochsn
Xs cao hochsn
 
Luong giac chuong 8
Luong giac chuong 8Luong giac chuong 8
Luong giac chuong 8
 
So phuc va cac bai toan lien quan 2
So phuc va cac bai toan lien quan 2So phuc va cac bai toan lien quan 2
So phuc va cac bai toan lien quan 2
 
10 dangtichphanhaygap thanhtung
10 dangtichphanhaygap thanhtung10 dangtichphanhaygap thanhtung
10 dangtichphanhaygap thanhtung
 
Viet pt-mat-phang-nt long - www.mathvn.com
Viet pt-mat-phang-nt long - www.mathvn.comViet pt-mat-phang-nt long - www.mathvn.com
Viet pt-mat-phang-nt long - www.mathvn.com
 
Tuyen tap cac bai toan hinh hoc giai tich trong khong gian (luyen thi dai hoc)
Tuyen tap cac bai toan hinh hoc giai tich trong khong gian (luyen thi dai hoc)Tuyen tap cac bai toan hinh hoc giai tich trong khong gian (luyen thi dai hoc)
Tuyen tap cac bai toan hinh hoc giai tich trong khong gian (luyen thi dai hoc)
 
[Vnmath.com] pt-bpt-hpt-mu-logarit-dh-2002-2011
[Vnmath.com] pt-bpt-hpt-mu-logarit-dh-2002-2011[Vnmath.com] pt-bpt-hpt-mu-logarit-dh-2002-2011
[Vnmath.com] pt-bpt-hpt-mu-logarit-dh-2002-2011
 
ham-so-on-thi-dh-huynh-bao-toan
 ham-so-on-thi-dh-huynh-bao-toan ham-so-on-thi-dh-huynh-bao-toan
ham-so-on-thi-dh-huynh-bao-toan
 
Phan loai cac dang toan dai so to hop
Phan loai cac dang toan dai so to hopPhan loai cac dang toan dai so to hop
Phan loai cac dang toan dai so to hop
 
Toan daisotohop-chuong5(2)
Toan daisotohop-chuong5(2)Toan daisotohop-chuong5(2)
Toan daisotohop-chuong5(2)
 
Toan d dh_2011
Toan d dh_2011Toan d dh_2011
Toan d dh_2011
 
Luonggiac chuong5
Luonggiac chuong5Luonggiac chuong5
Luonggiac chuong5
 
Tich phan phamkimchung-www.mathvn.com
Tich phan phamkimchung-www.mathvn.comTich phan phamkimchung-www.mathvn.com
Tich phan phamkimchung-www.mathvn.com
 
[Vnmath.com] 40 bai ham so chon loc nam 2013
[Vnmath.com] 40 bai ham so chon loc nam 2013[Vnmath.com] 40 bai ham so chon loc nam 2013
[Vnmath.com] 40 bai ham so chon loc nam 2013
 
Luonggiac chuong2
Luonggiac chuong2Luonggiac chuong2
Luonggiac chuong2
 
4 khao sat-ham_so_www.mathvn.com
4 khao sat-ham_so_www.mathvn.com4 khao sat-ham_so_www.mathvn.com
4 khao sat-ham_so_www.mathvn.com
 

Ähnlich wie Tuyen tap cac bai toan va phuong phap giai pt va bpt vo ty

04 phuong trinh mu p4
04 phuong trinh mu p404 phuong trinh mu p4
04 phuong trinh mu p4Huynh ICT
 
Phuong trinh vo ty
Phuong trinh vo tyPhuong trinh vo ty
Phuong trinh vo tytututhoi1234
 
Phương pháp giải phương trình, bất phương trình mũ
Phương pháp giải phương trình, bất phương trình mũPhương pháp giải phương trình, bất phương trình mũ
Phương pháp giải phương trình, bất phương trình mũLinh Nguyễn
 
Pt bpt-mu-loga-phan1
Pt bpt-mu-loga-phan1Pt bpt-mu-loga-phan1
Pt bpt-mu-loga-phan1thoang thoang
 
04 nguyen ham cua ham huu ti p3
04 nguyen ham cua ham huu ti p304 nguyen ham cua ham huu ti p3
04 nguyen ham cua ham huu ti p3Huynh ICT
 
1.3 bien luan_pt_bang_do_thi
1.3 bien luan_pt_bang_do_thi1.3 bien luan_pt_bang_do_thi
1.3 bien luan_pt_bang_do_thivanthuan1982
 
04 phuong trinh mu p1
04 phuong trinh mu p104 phuong trinh mu p1
04 phuong trinh mu p1Huynh ICT
 
Đáp Án Các Đề Thi Thử Toán 11 HK2
Đáp Án Các Đề Thi Thử Toán 11 HK2Đáp Án Các Đề Thi Thử Toán 11 HK2
Đáp Án Các Đề Thi Thử Toán 11 HK2thithanh2727
 
Tích phân hàm phân thức hữu tỷ (part 1)
Tích phân hàm phân thức hữu tỷ (part 1)Tích phân hàm phân thức hữu tỷ (part 1)
Tích phân hàm phân thức hữu tỷ (part 1)Oanh MJ
 
Tuyen tap cac bai toan va phuong phap giai pt va bpt vo ty
Tuyen tap cac bai toan va phuong phap giai  pt va bpt vo ty Tuyen tap cac bai toan va phuong phap giai  pt va bpt vo ty
Tuyen tap cac bai toan va phuong phap giai pt va bpt vo ty Huynh ICT
 
Bai mu-logarit
Bai mu-logaritBai mu-logarit
Bai mu-logaritnaovichet
 
10 kithuatgiaiphuongtrinhvoti thanhtung
10 kithuatgiaiphuongtrinhvoti thanhtung10 kithuatgiaiphuongtrinhvoti thanhtung
10 kithuatgiaiphuongtrinhvoti thanhtungHuynh ICT
 
72 hệ phương trình
72 hệ phương trình72 hệ phương trình
72 hệ phương trìnhtuituhoc
 
07 bat phuong trinh mu p1
07 bat phuong trinh mu p107 bat phuong trinh mu p1
07 bat phuong trinh mu p1Huynh ICT
 
06 ki thuat dong nhat tim nguyen ham
06 ki thuat dong nhat tim nguyen ham06 ki thuat dong nhat tim nguyen ham
06 ki thuat dong nhat tim nguyen hamHuynh ICT
 
100 bt về phương trình và hệ pt
100 bt về phương trình và hệ pt100 bt về phương trình và hệ pt
100 bt về phương trình và hệ ptDũng Bùi
 
Cực trị-hàm-đa-thức
Cực trị-hàm-đa-thứcCực trị-hàm-đa-thức
Cực trị-hàm-đa-thứcvanthuan1982
 
Tiet 30 ds 8 phep tru cac phan thuc dai so
Tiet 30 ds 8   phep tru cac phan thuc dai soTiet 30 ds 8   phep tru cac phan thuc dai so
Tiet 30 ds 8 phep tru cac phan thuc dai sohuynhngocquynhtan
 

Ähnlich wie Tuyen tap cac bai toan va phuong phap giai pt va bpt vo ty (20)

04 phuong trinh mu p4
04 phuong trinh mu p404 phuong trinh mu p4
04 phuong trinh mu p4
 
Phuong trinh vo ty
Phuong trinh vo tyPhuong trinh vo ty
Phuong trinh vo ty
 
Phương pháp giải phương trình, bất phương trình mũ
Phương pháp giải phương trình, bất phương trình mũPhương pháp giải phương trình, bất phương trình mũ
Phương pháp giải phương trình, bất phương trình mũ
 
Pt bpt-mu-loga-phan1
Pt bpt-mu-loga-phan1Pt bpt-mu-loga-phan1
Pt bpt-mu-loga-phan1
 
04 nguyen ham cua ham huu ti p3
04 nguyen ham cua ham huu ti p304 nguyen ham cua ham huu ti p3
04 nguyen ham cua ham huu ti p3
 
1.3 bien luan_pt_bang_do_thi
1.3 bien luan_pt_bang_do_thi1.3 bien luan_pt_bang_do_thi
1.3 bien luan_pt_bang_do_thi
 
04 phuong trinh mu p1
04 phuong trinh mu p104 phuong trinh mu p1
04 phuong trinh mu p1
 
Đáp Án Các Đề Thi Thử Toán 11 HK2
Đáp Án Các Đề Thi Thử Toán 11 HK2Đáp Án Các Đề Thi Thử Toán 11 HK2
Đáp Án Các Đề Thi Thử Toán 11 HK2
 
Bpt mu-logarit-2
Bpt mu-logarit-2Bpt mu-logarit-2
Bpt mu-logarit-2
 
Tích phân hàm phân thức hữu tỷ (part 1)
Tích phân hàm phân thức hữu tỷ (part 1)Tích phân hàm phân thức hữu tỷ (part 1)
Tích phân hàm phân thức hữu tỷ (part 1)
 
Tuyen tap cac bai toan va phuong phap giai pt va bpt vo ty
Tuyen tap cac bai toan va phuong phap giai  pt va bpt vo ty Tuyen tap cac bai toan va phuong phap giai  pt va bpt vo ty
Tuyen tap cac bai toan va phuong phap giai pt va bpt vo ty
 
Bai mu-logarit
Bai mu-logaritBai mu-logarit
Bai mu-logarit
 
200 logarit + giai
200 logarit + giai200 logarit + giai
200 logarit + giai
 
10 kithuatgiaiphuongtrinhvoti thanhtung
10 kithuatgiaiphuongtrinhvoti thanhtung10 kithuatgiaiphuongtrinhvoti thanhtung
10 kithuatgiaiphuongtrinhvoti thanhtung
 
72 hệ phương trình
72 hệ phương trình72 hệ phương trình
72 hệ phương trình
 
07 bat phuong trinh mu p1
07 bat phuong trinh mu p107 bat phuong trinh mu p1
07 bat phuong trinh mu p1
 
06 ki thuat dong nhat tim nguyen ham
06 ki thuat dong nhat tim nguyen ham06 ki thuat dong nhat tim nguyen ham
06 ki thuat dong nhat tim nguyen ham
 
100 bt về phương trình và hệ pt
100 bt về phương trình và hệ pt100 bt về phương trình và hệ pt
100 bt về phương trình và hệ pt
 
Cực trị-hàm-đa-thức
Cực trị-hàm-đa-thứcCực trị-hàm-đa-thức
Cực trị-hàm-đa-thức
 
Tiet 30 ds 8 phep tru cac phan thuc dai so
Tiet 30 ds 8   phep tru cac phan thuc dai soTiet 30 ds 8   phep tru cac phan thuc dai so
Tiet 30 ds 8 phep tru cac phan thuc dai so
 

Tuyen tap cac bai toan va phuong phap giai pt va bpt vo ty

  • 1. Ph¬ng tr×nh , BÊt ph¬ng tr×nh v« tØ Bµi 1: Gi¶i ph¬ng tr×nh a) + = −3 3 1 2 2 1x x + = − = − ⇔ + = 3 3 33 1 2 2 1 2 1 1 2 x x y x y x - Ph¬ng tr×nh ®îc chuyÓn thµnh hÖ   =  = =   + = + = + =  − +   ⇔ ⇔ ⇔ = =   + = − = − − + + + =      − − + = = = 33 3 3 3 3 2 2 3 1 1 21 2 1 2 1 5 21 2 2( ) 2 0( ) 1 51 2 2 x y x y x yx y x y x y y x x y x y x xy y vn x y x y - VËy ph¬ng tr×nh ®· cho cã 3 nghiÖm. b) + − = + −2 2 1 1 (1 2 1 )x x x §S:x=1/2; x=1 c) − + − = − + − +2 ( 3 2 1) 4 9 2 3 5 2x x x x x §S: x=2. d) + − + + − = − − 1 ( 3)( 1) 4( 3) 3 3 x x x x x §S: = − = −1 13; 1 5x x e) − + − = − +2 2 1 1 2 2 4 ( )x x x x - Sö dông B§T Bunhia. f) + − − = −4 1 1 2x x x §S: x=0 Bµi 2: Gi¶i BPT: a) + − − ≤5 1 4 1 3x x x §S: x≥1/4 b) − − + − > − − 2 2( 16) 7 3 3 3 x x x x x §K  − ≥ ⇔ ≥ − > 2 16 0 4 3 0 x x x - BiÕn ®«Ø bÊt ph¬ng tr×nh vÒ d¹ng − + − > − ⇔ − > − − < > − ≥⇔ ⇔ ⇔ > −  − < ≤  − > − 2 2 2 2 2( 16) 3 7 2( 16) 10 2 10 2 0 5 10 2 0 10 34. 10 34 5 2( 16) (10 2 ) x x x x x x x x x x x x - KÕt hîp §K ta cã nghiÖm cña BPT lµ > −10 34x . c) + − > −( 1)(4 ) 2x x x . 1
  • 2. d) − − < 2 1 1 4 3 x x . §K:  − ≤ < − ≥ ⇔  ≠  < ≤  2 1 0 1 4 0 2 10 0 2 x x x x - Thùc hiÖn phÐp nh©n liªn hîp ta thu ®îc BPT < + − ⇔ − > −  < − <    ≤− ≥ ⇔ ⇔ ⇔ ≤ − ≥  ≥ − > −    − > − 2 2 2 2 2 2 2 4 3(1 1 4 ) 3 1 4 4 3 3 44 3 0 1 1 4 0 1 2 24 3 0 3 9(1 4 ) (4 3) 4 9(1 4 ) (4 3) x x x x x x xx x x xx x x x . - KÕt hîp §K thu ®îc nghiÖm  − ≤ <   < ≤  1 0 2 1 0 2 x x C¸ch 2: - XÐt 2 TH: + Víi − ≤ < ⇔ − < −21 0. 1 4 1 3 2 x BPT x x + Víi < ≤ ⇔ − > −21 0 . 1 4 1 3 2 x BPT x x e) 2 2 5 10 1 7 2x x x x+ + ≥ − − §K: 2 5 2 5 5 5 10 1 0 5 2 5 5 x x x x  − − ≤ + + ≥ ⇔  − + ≥  - Víi §k ®ã 2 2 5 5 10 1 36 5 10 1x x x x− + + ≤ − + + + - §Æt 2 5 10 1; 0t x x t= + + ≥ . - §S: x≤-3 hoÆc x≥1. Bµi 3: T×m m ®Ó ph¬ng tr×nh sau cã nghiÖm: 2 2 1 1x x x x m+ + − − + = . Gi¶i: XÐt hµm sè 2 2 1 1y x x x x= + + − − + + MiÒn x¸c ®Þnh D=R . + §¹o hµm + − = − + + − + = ⇔ − + + = + − + − + > ⇔  − + + = + − + 2 2 2 2 2 2 2 2 2 1 2 1 ' 2 1 2 1 ' 0 (2 1) 1 (2 1) 1 (2 1)(2 1) 0 (vo nghiem) (2 1) ( 1) (2 1) ( 1) x x y x x x x y x x x x x x x x x x x x x x + y’(0)=1>0 nªn hµm sè §B 2
  • 3. + Giíi h¹n →−∞ →−∞ →+∞ = = − + + − − + = 2 2 2 lim lim 1 1 1 lim 1. x x x x y x x x x y + BBT x -∞ +∞ y’ + y 1 -1 VËy ph¬ng tr×nh cã nghiÖm khi vµ chØ khi -1<m<1. Bµi 4: T×m m ®Ó ph¬ng tr×nh sau cã nghiÖm thùc 2 1x x m+ = + Gi¶i: - §Æt 1; 0t x t= + ≥ . Ph¬ng tr×nh ®· cho trë thµnh: 2t=t2 -1+m m=-t2 +2t+1 - XÐt hµm sè y=-t2 +2t+1; t≥0; y’=-2t+2 x 0 1 +∞ y’ + 0 - y 2 1 -∞ - Theo yªu cÇu cña bµi to¸n ®êng th¼ng y=m c¾t §THS khi m≤2. Bµi 5: T×m m ®Ó ph¬ng tr×nh sau cã ®óng 2 nghiÖm d¬ng: 2 2 4 5 4x x m x x− + = + − . Gi¶i: - §Æt 2 2 2 ( ) 4 5; '( ) ; '( ) 0 2 4 5 x t f x x x f x f x x x x − = = − + = = ⇔ = − + . 3
  • 4. XÐt x>0 ta cã BBT: x 0 2 +∞ f’(x) - 0 + f(x) 5 +∞ 1 - Khi ®ã ph¬ng tr×nh ®· cho trë thµnh m=t2 +t-5 t2 +t-5-m=0 (1). - NÕu ph¬ng tr×nh (1) cã nghiÖm t1; t2 th× t1+ t2 =-1. Do ®ã (1) cã nhiÒu nhÊt 1 nghiÖm t≥1. - VËy ph¬ng tr×nh ®· cho cã ®óng 2 nghiÖm d¬ng khi vµ chØ khi ph¬ng tr×nh (1) cã ®óng 1 nghiÖm t (1; 5)∈ . - §Æt g(t)=t2 +t-5. Ta ®i t×m m ®Ó ph¬ng tr×nh g(t)=m cã ®óng 1 nghiÖm t (1; 5)∈ . f’(t)=2t+1>0 víi mäi t (1; 5)∈ . Ta cã BBT sau: t 1 5 g’(t) + g(t) 5 -3 Tõ BBT suy ra -3<m< 5 lµ c¸c gi¸ trÞ cÇn t×m. Bµi 6: X¸c ®Þnh m ®Ó ph¬ng tr×nh sau cã nghiÖm 2 2 4 2 2 ( 1 1 2) 2 1 1 1m x x x x x+ − − + = − + + − − . Gi¶i: - §iÒu kiÖn -1≤x≤1. §Æt 2 2 1 1t x x= + − − . - Ta cã 2 2 2 4 1 1 0; 0 0 2 2 1 2 2; 2 1 x x t t x t x t t x + ≥ − ⇒ ≥ = ⇔ = = − − ≤ ⇒ ≤ = ⇔ = ± - TËp gi¸ trÞ cña t lµ 0; 2    (t liªn tôc trªn ®o¹n [-1;1]). Ph¬ng tr×nh ®· cho trë thµnh: 2 2 2 ( 2) 2 (*) 2 t t m t t t m t − + + + = − + + ⇔ = + - XÐt 2 2 ( ) ;0 2. 2 t t f t t t − + + = ≤ ≤ + Ta cã f(t) liªn tôc trªn ®o¹n 0; 2    . Ph¬ng tr×nh ®· cho cã nghiÖm x khi vµ chØ khi ph¬ng tr×nh (*) cã nghiÖm t thuéc 0; 2    0; 2 0; 2 min ( ) max ( )f t m f t         ⇔ ≤ ≤ . - Ta cã 2 2 0; 2 0; 2 4 '( ) 0, 0; 2 ( ) 0; 2 . ( 2) Suy ra min ( ) ( 2) 2 1;max ( ) (0) 1 t t f t t f t NB t f t f f t f         − −    = ≤ ∀ ∈ ⇒    + = = − = = . - VËy 2 1 1.m− ≤ ≤ 4
  • 5. Bài 7: Tìm m để bất phương trình 3 1mx x m− − ≤ + (1) có nghiệm. Giải: Đặt 3; [0; )t x t= − ∈ +∞ . Bất phương trình trở thành: 2 2 2 1 ( 3) 1 ( 2) 1 2 t m t t m m t t m t + + − ≤ + ⇔ + ≤ + ⇔ ≤ + (2) (1)có nghiệm (2) có nghiệm t≥0  có ít nhất 1 điểm của ĐTHS y= 2 1 2 t t + + với t≥0 không ở phía dưới đường thẳng y=m. Xét y= 2 1 2 t t + + với t≥0 có 2 2 2 2 2 ' ( 2) t t y t − − + = + t 1 3− − 0 1 3− + + ∞ y’ - 0 + | + 0 - y 3 1 4 + Từ Bảng biến thiên ta có m≤ 3 1 4 + . Bài 8: Tìm m để phương trình 3 6 (3 )(6 )x x x x m+ + − − + − = có nghiệm. Giải: Đặt ( ) 3 6t f x x x= = + + − với [ 3;6]x∈ − thì 6 3 ' '( ) 2 (6 )(3 ) x x t f x x x − − + = = − + x -3 3/2 6 +∞ f’(x) ║ + 0 - ║ f(x) | 3 2 | 3 3 Vậy t [3;3 2]∈ . Phương trình (1) trở thành 2 2 9 9 2 2 2 t t t m t m − − = ⇔ − + + = (2). Phương trình (1) có nghiệm Phương trình (2) có nghiệm t [3;3 2]∈  đường thẳng y=m có điểm chung với đồ thị y= 2 9 2 2 t t− + + với t [3;3 2]∈ . Ta có y’=-t+1 nên có t 1 3 3 2 y’ + 0 - | - | y 3 9 3 2 2 − 5
  • 6. Bài 9: Cho bất phương trình 21 (4 )(2 ) (18 2 ) 4 x x a x x− + ≥ − + − . Tìm a để bất phương trình nghiệm đúng với mọi x∈[-2;4]. Giải: Đặt 2 (4 )(2 ) 2 8; [0;3]t x x x x t= − + = − + + ∈ . Bất phương trình trở thành: 2 21 (10 ) 4 10 4 t a t a t t≥ − + ⇔ ≥ − + .(2) (1)ghiệm  (2) có nghiệm mọi t∈[0;3] đường thẳng y=a nằm trên ĐTHS y=t2 -4t+10 với t∈[0;3] y’=2t-4; y’=0t=2 t 0 2 3 y’ | - 0 + | y 10 7 6 Vậy m≥10. Bài 10: Cho phương trình 4 2 2 2 ( 1)x x x m x+ + = + (1). Tìm m để phương trình có nghiệm. Giải: Phương trình đã cho tương đương 3 2 2 2 2 2 2 2 2 2 2 4( ) 4 ( 1) 4 2 2 4 2. ( ) 4 (1 ) (1 ) 1 1 x x x x x x x x m m m x x x x + + + + = ⇔ = ⇔ + = + + + + Đặt t= 2 2 1 x x+ ; t∈[-1;1]. Khi đó phương trình (1) trở thành 2t+t2 =4m. (1) có nghiệm  (2) có nghiệm t∈[-1;1] Xét hàm số y=f(t)=t2 +2t với t∈[-1;1]. Ta có f’(t)=2t+2≥0 với mọi t∈[-1;1]. t -1 1 f’ 0 + | f 3 -1 Từ BBT -1≤4m≤3 1 3 4 4 m⇔ − ≤ ≤ . 6
  • 7. HUYÊN ĐỀ : PHƯƠNG PHÁP GIẢI PHƯƠNG TRÌNH VÔ TỈ I. PHƯƠNG PHÁP BIẾN ĐỔI TƯƠNG ĐƯƠNG 1. Bình phương 2 vế của phương trình a) Phương pháp  Thông thường nếu ta gặp phương trình dạng : A B C D+ = + , ta thường bình phương 2 vế , điều đó đôi khi lại gặp khó khăn hãy giải ví dụ sau  ( )3 3 3 33 3 3 .A B C A B A B A B C+ = ⇒ + + + = và ta sử dụng phép thế : 3 3 A B C+ = ta được phương trình : 3 3 . .A B A B C C+ + = b) Ví dụ Bài 1. Giải phương trình sau : 3 3 1 2 2 2x x x x+ + + = + + Giải: Đk 0x ≥ Bình phương 2 vế không âm của phương trình ta được: ( ) ( ) ( )1 3 3 1 2 2 1x x x x x+ + + = + + , để giải phương trình này dĩ nhiên là không khó nhưng hơi phức tạp một chút . Phương trình giải sẽ rất đơn giản nếu ta chuyển vế phương trình : 3 1 2 2 4 3x x x x+ − + = − + Bình phương hai vế ta có : 2 2 6 8 2 4 12 1x x x x x+ + = + ⇔ = Thử lại x=1 thỏa  Nhận xét : Nếu phương trình : ( ) ( ) ( ) ( )f x g x h x k x+ = + Mà có : ( ) ( ) ( ) ( )f x h x g x k x+ = + , thì ta biến đổi phương trình về dạng : ( ) ( ) ( ) ( )f x h x k x g x− = − sau đó bình phương ,giải phương trình hệ quả Bài 2. Giải phương trình sau : 3 21 1 1 3 3 x x x x x x + + + = − + + + + Giải: Điều kiện : 1x ≥ − Bình phương 2 vế phương trình ? Nếu chuyển vế thì chuyển như thế nào? Ta có nhận xét : 3 21 . 3 1. 1 3 x x x x x x + + = − + + + , từ nhận xét này ta có lời giải như sau : 3 21 (2) 3 1 1 3 x x x x x x + ⇔ − + = − + − + + Bình phương 2 vế ta được: 3 2 2 1 31 1 2 2 0 3 1 3 xx x x x x x x  = −+ = − − ⇔ − − = ⇔  + = + Thử lại : 1 3, 1 3x x= − = + l nghiệm Qua lời giải trên ta có nhận xét : Nếu phương trình : ( ) ( ) ( ) ( )f x g x h x k x+ = + Mà có : ( ) ( ) ( ) ( ). .f x h x k x g x= thì ta biến đổi ( ) ( ) ( ) ( )f x h x k x g x− = − 2. Trục căn thức 2.1. Trục căn thức để xuất hiện nhân tử chung a) Phương pháp Một số phương trình vô tỉ ta có thể nhẩm được nghiệm 0x như vậy phương trình luôn đưa về được dạng tích ( ) ( )0 0x x A x− = ta có thể giải phương trình ( ) 0A x = hoặc chứng minh ( ) 0A x = vô nghiệm , chú ý điều kiện của nghiệm của phương trình để ta có thể đánh gía ( ) 0A x = vô nghiệm 7
  • 8. b) Ví dụ Bài 1 . Giải phương trình sau : ( )2 2 2 2 3 5 1 2 3 1 3 4x x x x x x x− + − − = − − − − + Giải: Ta nhận thấy : ( ) ( ) ( )2 2 3 5 1 3 3 3 2 2x x x x x− + − − − = − − v ( ) ( ) ( )2 2 2 3 4 3 2x x x x− − − + = − Ta có thể trục căn thức 2 vế : ( ) 2 22 2 2 4 3 6 2 3 43 5 1 3 1 x x x x xx x x x − + − = − + − +− + + − + Dể dàng nhận thấy x=2 là nghiệm duy nhất của phương trình . Bài 2. Giải phương trình sau (OLYMPIC 30/4 đề nghị) : 2 2 12 5 3 5x x x+ + = + + Giải: Để phương trình có nghiệm thì : 2 2 5 12 5 3 5 0 3 x x x x+ − + = − ≥ ⇔ ≥ Ta nhận thấy : x=2 là nghiệm của phương trình , như vậy phương trình có thể phân tích về dạng ( ) ( )2 0x A x− = , để thực hiện được điều đó ta phải nhóm , tách như sau : ( ) ( ) 2 2 2 2 2 2 2 2 4 4 12 4 3 6 5 3 3 2 12 4 5 3 2 1 2 3 0 2 12 4 5 3 x x x x x x x x x x x x x x − − + − = − + + − ⇔ = − + + + + +  + + ⇔ − − − = ⇔ = ÷ + + + +  Dễ dàng chứng minh được : 2 2 2 2 5 3 0, 312 4 5 3 x x x x x + + − − < ∀ > + + + + Bài 3. Giải phương trình : 2 33 1 1x x x− + = − Giải :Đk 3 2x ≥ Nhận thấy x=3 là nghiệm của phương trình , nên ta biến đổi phương trình ( ) ( ) ( ) ( )2 2 33 2 32 233 3 3 93 1 2 3 2 5 3 1 2 51 2 1 4 x x xx x x x x xx x   − + ++  − − + − = − − ⇔ − + =  − +− + − +   Ta chứng minh : ( ) ( ) 222 2 23 33 3 3 1 1 2 1 2 1 4 1 1 3 x x x x x + + + = + < − + − + − + + 2 3 3 9 2 5 x x x + + < − + Vậy pt có nghiệm duy nhất x=3 2.2. Đưa về “hệ tạm “ a) Phương pháp  Nếu phương trình vô tỉ có dạng A B C+ = , mà : A B Cα− = ở dây C có thể là hàng số ,có thể là biểu thức của x. Ta có thể giải như sau : A B C A B A B α − = ⇒ − = − , khi đĩ ta có hệ: 2 A B C A C A B α α  + = ⇒ = + − = b) Ví dụ Bài 4. Giải phương trình sau : 2 2 2 9 2 1 4x x x x x+ + + − + = + Giải: Ta thấy : ( ) ( ) ( )2 2 2 9 2 1 2 4x x x x x+ + − − + = + 4x = − không phải là nghiệm Xét 4x ≠ − Trục căn thức ta có : 2 2 2 2 2 8 4 2 9 2 1 2 2 9 2 1 x x x x x x x x x x + = + ⇒ + + − − + = + + − − + 8
  • 9. Vậy ta có hệ: 2 2 2 2 2 0 2 9 2 1 2 2 2 9 6 8 2 9 2 1 4 7 x x x x x x x x xx x x x x = + + − − + = ⇒ + + = + ⇔  =+ + + − + = +  Thử lại thỏa; vậy phương trình có 2 nghiệm : x=0 v x= 8 7 Bài 5. Giải phương trình : 2 2 2 1 1 3x x x x x+ + + − + = Ta thấy : ( ) ( )2 2 2 2 1 1 2x x x x x x+ + − − + = + , như vậy không thỏa mãn điều kiện trên. Ta có thể chia cả hai vế cho x và đặt 1 t x = thì bài toán trở nên đơn giản hơn Bài tập đề nghị Giải các phương trình sau : ( )2 2 3 1 3 1x x x x+ + = + + 4 3 10 3 2x x− − = − (HSG Toàn Quốc 2002) ( ) ( ) ( ) ( )2 2 5 2 10x x x x x− − = + − − 23 4 1 2 3x x x+ = − + − 2 33 1 3 2 3 2x x x− + − = − 2 3 2 11 21 3 4 4 0x x x− + − − = (OLYMPIC 30/4-2007) 2 2 2 2 2 1 3 2 2 2 3 2x x x x x x x− + − − = + + + − + 2 2 2 16 18 1 2 4x x x x+ + + − = + 2 2 15 3 2 8x x x+ = − + + 3. Phương trình biến đổi về tích  Sử dụng đẳng thức ( ) ( )1 1 1 0u v uv u v+ = + ⇔ − − = ( ) ( ) 0au bv ab vu u b v a+ = + ⇔ − − = 2 2 A B= Bài 1. Giải phương trình : 233 3 1 2 1 3 2x x x x+ + + = + + + Giải: ( )( )3 3 0 1 1 2 1 0 1 x pt x x x = ⇔ + − + − = ⇔  = − Bi 2. Giải phương trình : 2 23 33 3 1x x x x x+ + = + + Giải: + 0x = , không phải là nghiệm + 0x ≠ , ta chia hai vế cho x: ( )3 3 33 3 1 1 1 1 1 1 0 1 x x x x x x x x  + + + = + + ⇔ − − = ⇔ = ÷   Bài 3. Giải phương trình: 2 3 2 1 2 4 3x x x x x x+ + + = + + + Giải: : 1dk x ≥ − pt ( )( ) 1 3 2 1 1 0 0 x x x x x = ⇔ + − + − = ⇔  = Bài 4. Giải phương trình : 4 3 4 3 x x x x + + = + Giải: Đk: 0x ≥ Chia cả hai vế cho 3x + : 2 4 4 4 1 2 1 0 1 3 3 3 x x x x x x x   + = ⇔ − = ⇔ = ÷ + + +   Dùng hằng đẳng thức 9
  • 10. Biến đổi phương trình về dạng : k k A B= Bài 1. Giải phương trình : 3 3x x x− = + Giải: Đk: 0 3x≤ ≤ khi đó pt đ cho tương đương : 3 2 3 3 0x x x+ + − = 3 3 1 10 10 1 3 3 3 3 x x −  ⇔ + = ⇔ = ÷   Bài 2. Giải phương trình sau : 2 2 3 9 4x x x+ = − − Giải: Đk: 3x ≥ − phương trình tương đương : ( ) 2 2 1 3 1 3 1 3 9 5 97 3 1 3 18 x x x x x xx x = + + = + + = ⇔ ⇔ − − =+ + = −  Bài 3. Giải phương trình sau : ( ) ( ) 22 332 3 9 2 2 3 3 2x x x x x+ + = + + Giải : pttt ( ) 3 3 3 2 3 0 1x x x⇔ + − = ⇔ = II. PHƯƠNG PHÁP ĐẶT ẦN PHỤ 1. Phương pháp đặt ẩn phụ thông thường  Đối với nhiều phương trình vô vô tỉ , để giải chúng ta có thể đặt ( )t f x= và chú ý điều kiện của t nếu phương trình ban đầu trở thành phương trình chứa một biến t quan trọng hơn ta có thể giải được phương trình đó theo t thì việc đặt phụ xem như “hoàn toàn ” .Nói chung những phương trình mà có thể đặt hoàn toàn ( )t f x= thường là những phương trình dễ . Bài 1. Giải phương trình: 2 2 1 1 2x x x x− − + + − = Điều kiện: 1x ≥ Nhận xét. 2 2 1. 1 1x x x x− − + − = Đặt 2 1t x x= − − thì phương trình có dạng: 1 2 1t t t + = ⇔ = Thay vào tìm được 1x = Bài 2. Giải phương trình: 2 2 6 1 4 5x x x− − = + Giải Điều kiện: 4 5 x ≥ − Đặt 4 5( 0)t x t= + ≥ thì 2 5 4 t x − = . Thay vào ta có phương trình sau: 4 2 2 4 210 25 6 2. ( 5) 1 22 8 27 0 16 4 t t t t t t t − + − − − = ⇔ − − + = 2 2 ( 2 7)( 2 11) 0t t t t⇔ + − − − = Ta tìm được bốn nghiệm là: 1,2 3,41 2 2; 1 2 3t t= − ± = ± Do 0t ≥ nên chỉ nhận các gái trị 1 31 2 2, 1 2 3t t= − + = + Từ đó tìm được các nghiệm của phương trình l: 1 2 2 3vaøx x= − = + Cách khác: Ta có thể bình phương hai vế của phương trình với điều kiện 2 2 6 1 0x x− − ≥ Ta được: 2 2 2 ( 3) ( 1) 0x x x− − − = , từ đó ta tìm được nghiệm tương ứng. Đơn giản nhất là ta đặt : 2 3 4 5y x− = + và đưa về hệ đối xứng (Xem phần dặt ẩn phụ đưa về hệ) Bài 3. Giải phương trình sau: 5 1 6x x+ + − = 10
  • 11. Điều kiện: 1 6x≤ ≤ Đặt 1( 0)y x y= − ≥ thì phương trình trở thnh: 2 4 2 5 5 10 20 0y y y y y+ + = ⇔ − − + = ( với 5)y ≤ 2 2 ( 4)( 5) 0y y y y⇔ + − − − = 1 21 1 17 , 2 2 (loaïi)y y + − + ⇔ = = Từ đó ta tìm được các giá trị của 11 17 2 x − = Bài 4. (THTT 3-2005) Giải phương trình sau : ( )( ) 2 2004 1 1x x x= + − − Giải: đk 0 1x≤ ≤ Đặt 1y x= − pttt ( ) ( )2 2 2 1 1002 0 1 0y y y y x⇔ − + − = ⇔ = ⇔ = Bài 5. Giải phương trình sau : 2 1 2 3 1x x x x x + − = + Giải: Điều kiện: 1 0x− ≤ < Chia cả hai vế cho x ta nhận được: 1 1 2 3x x x x + − = + Đặt 1 t x x = − , ta giải được. Bài 6. Giải phương trình : 2 4 23 2 1x x x x+ − = + Giải: 0x = không phải là nghiệm , Chia cả hai vế cho x ta được: 3 1 1 2x x x x   − + − = ÷   Đặt t= 3 1 x x − , Ta có : 3 2 0t t+ − = ⇔ 1 5 1 2 t x ± = ⇔ = Bài tập đề nghị Giải các phương trình sau 2 2 15 2 5 2 15 11x x x x− − = − + 2 ( 5)(2 ) 3 3x x x x+ − = + 2 (1 )(2 ) 1 2 2x x x x+ − = + − 2 2 17 17 9x x x x+ − + − = 2 3 2 1 4 9 2 3 5 2x x x x x− + − = − + − + 2 2 11 31x x+ + = 2 2 2 2 (1 ) 3 1 (1 ) 0nn n x x x+ + − + − = 2 (2004 )(1 1 )x x x= + − − ( 3 2)( 9 18) 168x x x x x+ + + + = 32 2 1 2 1 3x x− + − = Nhận xét : đối với cách đặt ẩn phụ như trên chúng ta chỉ giải quyết được một lớp bài đơn giản, đôi khi phương trình đối với t lại quá khó giải 2. Đặt ẩn phụ đưa về phương trình thuần nhất bậc 2 đối với 2 biến :  Chúng ta đã biết cách giải phương trình: 2 2 0u uv vα β+ + = (1) bằng cách Xét 0v ≠ phương trình trở thành : 2 0 u u v v α β     + + = ÷  ÷     0v = thử trực tiếp Các trường hợp sau cũng đưa về được (1)  ( ) ( ) ( ) ( ). .a A x bB x c A x B x+ =  2 2 u v mu nvα β+ = + 11
  • 12. Chúng ta hãy thay các biểu thức A(x) , B(x) bởi các biểu thức vô tỉ thì sẽ nhận được phương trình vô tỉ theo dạng này . a) . Phương trình dạng : ( ) ( ) ( ) ( ). .a A x bB x c A x B x+ = Như vậy phương trình ( ) ( )Q x P xα= có thể giải bằng phương pháp trên nếu ( ) ( ) ( ) ( ) ( ) ( ) .P x A x B x Q x aA x bB x  =  = + Xuất phát từ đẳng thức : ( ) ( )3 2 1 1 1x x x x+ = + − + ( ) ( )( )4 2 4 2 2 2 2 1 2 1 1 1x x x x x x x x x+ + = + + − = + + − + ( )( )4 2 2 1 2 1 2 1x x x x x+ = − + + + ( )( )4 2 2 4 1 2 2 1 2 2 1x x x x x+ = − + + + Hãy tạo ra những phương trình vô tỉ dạng trên ví dụ như: 2 4 4 2 2 4 1x x x− + = + Để có một phương trình đẹp , chúng ta phải chọn hệ số a,b,c sao cho phương trình bậc hai 2 0at bt c+ − = giải “ nghiệm đẹp” Bài 1. Giải phương trình : ( )2 3 2 2 5 1x x+ = + Giải: Đặt 2 1, 1u x v x x= + = − + Phương trình trở thành : ( )2 2 2 2 5 1 2 u v u v uv u v = + = ⇔  =  Tìm được: 5 37 2 x ± = Bài 2. Giải phương trình : 2 4 23 3 1 1 3 x x x x− + = − + + Bài 3: giải phương trình sau : 2 3 2 5 1 7 1x x x+ − = − Giải: Đk: 1x ≥ Nhận xt : Ta viết ( ) ( ) ( ) ( )2 2 1 1 7 1 1x x x x x xα β− + + + = − + + Đồng nhất thức ta được: ( ) ( ) ( ) ( )2 2 3 1 2 1 7 1 1x x x x x x− + + + = − + + Đặt 2 1 0, 1 0u x v x x= − ≥ = + + > , ta được: 9 3 2 7 1 4 v u u v uv v u = + = ⇔  =  Ta được : 4 6x = ± Bài 4. Giải phương trình : ( ) 33 2 3 2 2 6 0x x x x− + + − = Giải: Nhận xét : Đặt 2y x= + ta hãy biến pt trên về phương trình thuần nhất bậc 3 đối với x và y : 3 2 3 3 2 3 3 2 6 0 3 2 0 2 x y x x y x x xy y x y = − + − = ⇔ − + = ⇔  = − Pt có nghiệm : 2, 2 2 3x x= = − b).Phương trình dạng : 2 2 u v mu nvα β+ = + Phương trình cho ở dạng này thường khó “phát hiện “ hơn dạng trên , nhưg nếu ta bình phương hai vế thì đưa về được dạng trên. Bài 1. giải phương trình : 2 2 4 2 3 1 1x x x x+ − = − + Giải: 12
  • 13. Ta đặt : 2 2 1 u x v x  =  = − khi đó phương trình trở thành : 2 2 3u v u v+ = − Bài 2.Giải phương trình sau : 2 2 2 2 1 3 4 1x x x x x+ + − = + + Giải Đk 1 2 x ≥ . Bình phương 2 vế ta có : ( )( ) ( )( ) ( ) ( )2 2 2 2 2 2 1 1 2 2 1 2 2 1x x x x x x x x x x+ − = + ⇔ + − = + − − Ta có thể đặt : 2 2 2 1 u x x v x  = +  = − khi đó ta có hệ : 2 2 1 5 2 1 5 2 u v uv u v u v  − = = − ⇔  + =  Do , 0u v ≥ . ( )21 5 1 5 2 2 1 2 2 u v x x x + + = ⇔ + = − Bài 3. giải phương trình : 2 2 5 14 9 20 5 1x x x x x− + − − − = + Giải: Đk 5x ≥ . Chuyển vế bình phương ta được: ( )( )2 2 2 5 2 5 20 1x x x x x− + = − − + Nhận xét : không tồn tại số ,α β để : ( ) ( )2 2 2 5 2 20 1x x x x xα β− + = − − + + vậy ta không thể đặt 2 20 1 u x x v x  = − −  = + . Nhưng may mắn ta có : ( )( ) ( ) ( ) ( ) ( ) ( )2 2 20 1 4 5 1 4 4 5x x x x x x x x x− − + = + − + = + − − Ta viết lại phương trình: ( ) ( )2 2 2 4 5 3 4 5 ( 4 5)( 4)x x x x x x− − + + = − − + . Đến đây bài toán được giải quyết . Các em hãy tự sáng tạo cho mình những phương trình vô tỉ “đẹp “ theo cách trên 3. Phương pháp đặt ẩn phụ không hoàn toàn  Từ những phương trình tích ( )( )1 1 1 2 0x x x+ − + − + = ,( )( )2 3 2 3 2 0x x x x+ − + − + = Khai triển và rút gọn ta sẽ được những phương trình vô tỉ không tầm thường chút nào, độ khó của phương trình dạng này phụ thuộc vào phương trình tích mà ta xuất phát . Từ đó chúng ta mới đi tìm cách giải phương trình dạng này .Phương pháp giải được thể hiện qua các ví dụ sau . Bài 1. Giải phương trình : ( )2 2 2 3 2 1 2 2x x x x+ − + = + + Giải: 2 2t x= + , ta có : ( )2 3 2 3 3 0 1 t t x t x t x = − + − + = ⇔  = − Bài 2. Giải phương trình : ( ) 2 2 1 2 3 1x x x x+ − + = + Giải: Đặt : 2 2 3, 2t x x t= − + ≥ Khi đó phương trình trở thnh : ( ) 2 1 1x t x+ = + ( )2 1 1 0x x t⇔ + − + = 13
  • 14. Bây giờ ta thêm bớt , để được phương trình bậc 2 theo t có ∆ chẵn : ( ) ( ) ( ) ( )2 2 2 2 3 1 2 1 0 1 2 1 0 1 t x x x t x t x t x t x = − + − + + − = ⇔ − + + − = ⇔  = − Từ một phương trình đơn giản : ( )( )1 2 1 1 2 1 0x x x x− − + − − + + = , khai triển ra ta sẽ được pt sau Bài 3. Giải phương trình sau : 2 4 1 1 3 2 1 1x x x x+ − = + − + − Giải: Nhận xét : đặt 1t x= − , pttt: 4 1 3 2 1x x t t x+ = + + + (1) Ta rút 2 1x t= − thay vào thì được pt: ( ) ( )2 3 2 1 4 1 1 0t x t x− + + + + − = Nhưng không có sự may mắn để giải được phương trình theo t ( ) ( ) 2 2 1 48 1 1x x∆ = + + − + − không có dạng bình phương . Muốn đạt được mục đích trên thì ta phải tách 3x theo ( ) ( ) 2 2 1 , 1x x− + Cụ thể như sau : ( ) ( )3 1 2 1x x x= − − + + thay vào pt (1) ta được: Bài 4. Giải phương trình: 2 2 2 4 4 2 9 16x x x+ + − = + Giải . Bình phương 2 vế phương trình: ( ) ( ) ( )2 2 4 2 4 16 2 4 16 2 9 16x x x x+ + − + − = + Ta đặt : ( )2 2 4 0t x= − ≥ . Ta được: 2 9 16 32 8 0x t x− − + = Ta phải tách ( ) ( )2 2 2 9 2 4 9 2 8x x xα α α= − + + − làm sao cho t∆ có dạng chính phương . Nhận xét : Thông thường ta chỉ cần nhóm sao cho hết hệ số tự do thì sẽ đạt được mục đích 4. Đặt nhiều ẩn phụ đưa về tích  Xuất phát từ một số hệ “đại số “ đẹp chúng ta có thể tạo ra được những phương trình vô tỉ mà khi giải nó chúng ta lại đặt nhiều ẩn phụ và tìm mối quan hệ giữa các ẩn phụ để đưa về hệ Xuất phát từ đẳng thức ( ) ( ) ( ) ( ) 3 3 3 3 3a b c a b c a b b c c a+ + = + + + + + + , Ta có ( ) ( )( ) ( ) 33 3 3 0a b c a b c a b a c b c+ + = + + ⇔ + + + = Từ nhận xét này ta có thể tạo ra những phương trình vô tỉ có chứa căn bậc ba . 2 23 33 7 1 8 8 1 2x x x x x+ − − − + − + = 3 3 3 3 3 1 5 2 9 4 3 0x x x x+ + − + − − − = Bài 1. Giải phương trình : 2 . 3 3 . 5 5 . 2x x x x x x x= − − + − − + − − Giải : 2 3 5 u x v x w x  = −  = −  = − , ta có : ( )( ) ( )( ) ( )( ) 2 2 2 22 3 3 5 5 u v u wu uv vw wu v uv vw wu u v v w w uv vw wu v w u w  + + = − = + +  − = + + ⇔ + + =    − = + + + + =  , giải hệ ta được: 30 239 60 120 u x= ⇔ = Bài 2. Giải phương trình sau : 2 2 2 2 2 1 3 2 2 2 3 2x x x x x x x− + − − = + + + − + 14
  • 15. Giải . Ta đặt : 2 2 2 2 2 1 3 2 2 2 3 2 a x b x x c x x d x x  = −   = − −  = + +  = − + , khi đó ta có : 2 2 2 2 2 a b c d x a b c d + = + ⇔ = − − = − Bài 3. Giải các phương trình sau 1) 2 2 4 5 1 2 1 9 3x x x x x+ + − − + = − 2) ( ) ( ) ( ) 3 3 244 44 1 1 1 1x x x x x x x x+ − + − = − + + − 5. Đặt ẩn phụ đưa về hệ: 5.1 Đặt ẩn phụ đưa về hệ thông thường  Đặt ( ) ( ),u x v xα β= = và tìm mối quan hệ giữa ( )xα và ( )xβ từ đó tìm được hệ theo u,v Bài 1. Giải phương trình: ( )3 33 3 25 25 30x x x x− + − = Đặt 3 3 3 3 35 35y x x y= − ⇒ + = Khi đó phương trình chuyển về hệ phương trình sau: 3 3 ( ) 30 35 xy x y x y + =  + = , giải hệ này ta tìm được ( ; ) (2;3) (3;2)x y = = . Tức là nghiệm của phương trình là {2;3}x∈ Bài 2. Giải phương trình: 4 4 1 2 1 2 x x− − + = Điều kiện: 0 2 1x≤ ≤ − Đặt 4 4 2 1 0 2 1,0 2 1 x u u v x v  − − = ⇒ ≤ ≤ − ≤ ≤ − = Ta đưa về hệ phương trình sau: 4 4 2 2 4 4 4 1 1 2 2 1 2 1 2 1 2 u v u v u v v v  = − + =  ⇔    + = − − + = −  ÷  Giải phương trình thứ 2: 2 2 2 4 1 ( 1) 0 2 v v   + − + = ÷   , từ đó tìm ra v rồi thay vào tìm nghiệm của phương trình. Bài 3. Giải phương trình sau: 5 1 6x x+ + − = Điều kiện: 1x ≥ Đặt 1, 5 1( 0, 0)a x b x a b= − = + − ≥ ≥ thì ta đưa về hệ phương trình sau: 2 2 5 ( )( 1) 0 1 0 1 5 a b a b a b a b a b b a  + = → + − + = ⇒ − + = ⇒ = − − = Vậy 11 17 1 1 5 1 1 5 2 x x x x x − − + = + − ⇔ − = − ⇒ = Bài 8. Giải phương trình: 6 2 6 2 8 35 5 x x x x − + + = − + Giải Điều kiện: 5 5x− < < 15
  • 16. Đặt ( )5 , 5 0 , 10u x v y u v= − = − < < . Khi đó ta được hệ phương trình: 22 2 ( ) 10 210 2 44 4 8 ( ) 12( ) 33 u v uvu v u vu z uvu v  + = ++ =   ⇔    + − =− − + + =  ÷     5.2 Xây dựng phương trình vô tỉ từ hệ đối xứng loại II  Ta hãy đi tìm nguồn gốc của những bài toán giải phương trình bằng cách đưa về hệ đối xứng loại II  Ta xét một hệ phương trình đối xứng loại II sau : ( ) ( ) 2 2 1 2 (1) 1 2 (2) x y y x  + = +  + = + việc giải hệ này thì đơn giản Bây giời ta sẽ biến hệ thành phương trình bằng cách đặt ( )y f x= sao cho (2) luôn đúng , 2 1y x= + − , khi đó ta có phương trình : ( ) 2 2 1 ( 2 1) 1 2 2x x x x x+ = + − + ⇔ + = + Vậy để giải phương trình : 2 2 2x x x+ = + ta đặt lại như trên và đưa về hệ Bằng cách tương tự xét hệ tổng quát dạng bậc 2 : ( ) ( ) 2 2 x ay b y ax b α β α β  + = +  + = + , ta sẽ xây dựng được phương trình dạng sau : đặt y ax bα β+ = + , khi đó ta có phương trình : ( ) 2 a x ax b b β α β α α + = + + − Tương tự cho bậc cao hơn : ( ) n na x ax b b β α β α α + = + + − Tóm lại phương trình thường cho dưới dạng khai triển ta phải viết về dạng : ( ) ' ' n n x p a x bα β γ+ = + + v đặt n y ax bα β+ = + để đưa về hệ , chú ý về dấu của α ??? Việc chọn ;α β thông thường chúng ta chỉ cần viết dưới dạng :( ) ' ' n n x p a x bα β γ+ = + + là chọn được. Bài 1. Giải phương trình: 2 2 2 2 1x x x− = − Điều kiện: 1 2 x ≥ Ta có phương trình được viết lại là: 2 ( 1) 1 2 2 1x x− − = − Đặt 1 2 1y x− = − thì ta đưa về hệ sau: 2 2 2 2( 1) 2 2( 1) x x y y y x  − = −  − = − Trừ hai vế của phương trình ta được ( )( ) 0x y x y− + = Giải ra ta tìm được nghiệm của phương trình là: 2 2x = + Bài 6. Giải phương trình: 2 2 6 1 4 5x x x− − = + Giải Điều kiện 5 4 x ≥ − Ta biến đổi phương trình như sau: 2 2 4 12 2 2 4 5 (2 3) 2 4 5 11x x x x x− − = + ⇔ − = + + Đặt 2 3 4 5y x− = + ta được hệ phương trình sau: 2 2 (2 3) 4 5 ( )( 1) 0 (2 3) 4 5 x y x y x y y x  − = + ⇒ − + − = − = + Với 2 3 4 5 2 3x y x x x= ⇒ − = + ⇒ = + Với 1 0 1 1 2x y y x x+ − = ⇒ = − → = − 16
  • 17. Kết luận: Nghiệm của phương trình là {1 2; 1 3}− + Các em hãy xây dựng một sồ hệ dạng này ?  Dạng hệ gần đối xứng Ta xt hệ sau : 2 2 (2 3) 2 1 (1) (2 3) 3 1 x y x y x  − = + +  − = + đây không phải là hệ đối xứng loại 2 nhưng chúng ta vẫn giải hệ được , và từ hệ này chúng ta xây dưng được bài toán phương trình sau : Bài 1 . Giải phương trình: 2 4 5 13 3 1 0x x x+ − + + = Nhận xét : Nếu chúng ta nhóm như những phương trình trước : 2 13 33 2 3 1 4 4 x x   − = + − ÷   Đặt 13 2 3 1 4 y x− = + thì chúng ta không thu được hệ phương trình mà chúng ta có thể giải được. Để thu được hệ (1) ta đặt : 3 1y xα β+ = + , chọn ,α β sao cho hệ chúng ta có thể giải được , (đối xứng hoặc gần đối xứng ) Ta có hệ : ( ) 2 2 2 2 22 2 3 1 0 (1)3 1 (*) 4 13 5 0 (2)4 13 5 y y xy x x x yx x y α αβ βα β α βα β   + − + − =+ = +  ⇔  − + + + =− + = − −  Để giải hệ trên thì ta lấy (1) nhân với k cộng với (2): và mong muốn của chúng ta là có nghiệm x y= Nên ta phải có : 2 2 2 3 1 4 13 5 α αβ β α β − − = = − + , ta chọn được ngay 2; 3α β= − = Ta có lời giải như sau : Điều kiện: 1 3 x ≥ − , Đặt 3 3 1 (2 3), ( ) 2 x y y+ = − − ≤ Ta có hệ phương trình sau: 2 2 (2 3) 2 1 ( )(2 2 5) 0 (2 3) 3 1 x y x x y x y y x  − = + + ⇒ − + − = − = + Với 15 97 8 x y x − = ⇒ = Với 11 73 2 2 5 0 8 x y x + + − = ⇒ = Kết luận: tập nghiệm của phương trình là: 15 97 11 73 ; 8 8  − +       Chú ý : khi đã làm quen, chúng ta có thể tìm ngay ;α β bằng cách viết lại phương trình ta viết lại phương trình như sau: 2 (2 3) 3 1 4x x x− = − + + + khi đó đặt 3 1 2 3x y+ = − + , nếu đặt 2 3 3 1y x− = + thì chúng ta không thu được hệ như mong muốn , ta thấy dấu của α cùng dấu với dấu trước căn. Một cách tổng quát . Xét hệ: ( ) . . (1) ( ) '. ' (2) f x A x B y m f y A x m = + +  = + để hệ có nghiệm x = y thì : A-A’=B và m=m’, Nếu từ (2) tìm được hàm ngược ( )y g x= thay vào (1) ta được phương trình Như vậy để xây dựng pt theo lối này ta cần xem xét để có hàm ngược và tìm được và hơn nữa hệ phải giải được. Một số phương trình được xây dựng từ hệ. Giải các phương trình sau 1) 2 4 13 5 3 1 0x x x− + + + = 4) 33 6 1 8 4 1x x x+ = − − 17
  • 18. 2) 2 4 13 5 3 1 0x x x− + + + = 3) 3 23 4 81 8 2 2 3 x x x x− = − + − 5) ( ) ( )215 30 4 2004 30060 1 1 2 x x x− = + + 6) 3 23 3 5 8 36 53 25x x x− = − + − Giải (3): Phương trình : ( ) 33 23 3 27 81 8 27 54 36 54 27 81 8 3 2 46x x x x x x⇔ − = − + − ⇔ − = − − Ta đặt : 3 3 2 81 8y x− = − Các em hãy xây dựng những phương trình dạng này ! III. PHƯƠNG PHÁP ĐÁNH GIÁ 1. Dùng hằng đẳng thức :  Từ những đánh giá bình phương : 2 2 0A B+ ≥ , ta xây dựng phương trình dạng 2 2 0A B+ = Từ phương trình ( ) ( ) 2 2 5 1 2 9 5 2 1 0x x x x− − + − − + − = ta khai triển ra có phương trình : ( )2 4 12 1 4 5 1 9 5x x x x x+ + − = − + − 2. Dùng bất đẳng thức  Một số phương trình được tạo ra từ dấu bằng của bất đẳng thức: A m B m ≥  ≤ nếu dấu bằng ỏ (1) và (2) cùng dạt được tại 0x thì 0x là nghiệm của phương trình A B= Ta có : 1 1 2x x+ + − ≤ Dấu bằng khi và chỉ khi 0x = và 1 1 2 1 x x + + ≥ + , dấu bằng khi và chỉ khi x=0. Vậy ta có phương trình: 1 1 2008 1 2008 1 1 x x x x − + + = + + + Đôi khi một số phương trình được tạo ra từ ý tưởng : ( ) ( ) A f x B f x  ≥  ≤ khi đó : ( ) ( ) A f x A B B f x  = = ⇔  =  Nếu ta đoán trước được nghiệm thì việc dùng bất đẳng thức dễ dàng hơn, nhưng có nhiều bài nghiệm là vô tỉ việc đoán nghiệm không được, ta vẫn dùng bất đẳng thức để đánh giá được Bài 1. Giải phương trình (OLYMPIC 30/4 -2007): 2 2 9 1 x x x + = + + Giải: Đk 0x ≥ Ta có : ( ) 2 2 22 2 1 2 2 1 9 11 1 x x x x xx x        + ≤ + + + = + ÷  ÷   ++ +      Dấu bằng 2 2 1 1 71 1 x x x ⇔ = ⇔ = + + Bài 2. Giải phương trình : 2 4 2 4 13 9 16x x x x− + + = Giải: Đk: 1 1x− ≤ ≤ Biến đổi pt ta có : ( ) 2 2 2 2 13 1 9 1 256x x x− + + = Áp dụng bất đẳng thức Bunhiacopxki: ( ) ( ) ( ) ( ) 2 2 2 2 2 2 13. 13. 1 3. 3. 3 1 13 27 13 13 3 3 40 16 10x x x x x− + + ≤ + − + + = − 18
  • 19. Áp dụng bất đẳng thức Côsi: ( ) 2 2 2 16 10 16 10 64 2 x x   − ≤ = ÷   Dấu bằng 2 2 2 2 2 1 51 3 2 10 16 10 5 xx x xx x   =+ − = ⇔ ⇔  = −= −  Bài 3. giải phương trình: 3` 2 4 3 8 40 8 4 4 0x x x x− − + − + = Ta chứng minh : 4 8 4 4 13x x+ ≤ + và ( ) ( ) 23 2 3 8 40 0 3 3 13x x x x x x− − + ≥ ⇔ − + ≥ + Bài tập đề nghị . Giải các phương trình sau 1 2 1 2 1 2 1 2 1 2 1 2 x x x x x x − + − + + = + + − 4 4 4 1 1 2 8x x x x+ − + − − = + 4 4 4 2 8 4 4 4 4x x x+ = + + − 4 33 16 5 6 4x x x+ = + 3` 2 4 3 8 40 8 4 4 0x x x x− − + − + = 3 3 4 2 8 64 8 28x x x x+ + − = − + 2 2 1 1 2 2 4x x x x   − + − = − + ÷   3. Xây dựng bài toán từ tính chất cực trị hình học 3.1 Dùng tọa độ của véc tơ  Trong mặt phẳng tọa độ Oxy, Cho các véc tơ: ( ) ( )1 1 2 2; , ;u x y v x y= = r r khi đó ta có  ( ) ( ) 2 2 2 2 2 2 1 2 1 2 1 1 2 2u v u v x x y y x y x y+ ≤ + ⇔ + + + ≤ + + + r r r r Dấu bằng xẩy ra khi và chỉ khi hai véc tơ ,u v r r cùng hướng 1 1 2 2 0 x y k x y ⇔ = = ≥ , chú ý tỉ số phải dương  . . .cos .u v u v u vα= ≤ r r r r r r , dấu bằng xẩy ra khi và chỉ khi cos 1 u vα = ⇔ ↑↑ r 3.2 Sử dụng tính chất đặc biệt về tam giác  Nếu tam giác ABC là tam giác đều , thì với mọi điểm M trên mặt phẳng tam giác, ta luôn có MA MB MC OA OB OC+ + ≥ + + với O là tâm của đường tròn .Dấu bằng xẩy ra khi và chỉ khi M O≡ .  Cho tam giác ABC có ba góc nhọn và điểm M tùy ý trong mặt mặt phẳng Thì MA+MB+MC nhỏ nhất khi điểm M nhìn các cạnh AB,BC,AC dưới cùng một góc 0 120 Bài tập 1) ( ) ( )2 2 2 2 2 1 2 3 1 1 2 3 1 1 3x x x x x x− + + − − + + + + + = 2) 2 2 4 5 10 50 5x x x x− + − − + = IV. PHƯƠNG PHÁP HÀM SỐ 1.Xây dựng phương trình vô tỉ dựa theo hàm đơn điệu  Dựa vào kết quả : “ Nếu ( )y f t= là hàm đơn điệu thì ( ) ( )f x f t x t= ⇔ = ” ta có thể xây dựng được những phương trình vô tỉ Xuất phát từ hàm đơn điệu : ( ) 3 2 2 1y f x x x= = + + mọi 0x ≥ ta xây dựng phương trình : ( ) ( ) ( ) 3 3 2 2 3 1 2 1 2 3 1 (3 1) 1f x f x x x x x= − ⇔ + + = − + − + , Rút gọn ta được phương trình 19
  • 20. ( )3 2 2 3 1 2 3 1 3 1x x x x x+ − + = − − Từ phương trình ( ) ( )1 3 1f x f x+ = − thì bài toán sẽ khó hơn ( ) ( )3 2 2 7 5 4 2 3 1 3 1x x x x x+ + + = − − Để gải hai bài toán trên chúng ta có thể làm như sau : Đặt 3 1y x= − khi đó ta có hệ : 3 2 3 2 2 7 5 4 2 3 1 x x x y x y  + + + =  − = cộng hai phương trình ta được: ( ) ( ) 3 2 2 1 1x x+ + + = 3 2 2y y+ Hãy xây dựng những hàm đơn điệu và những bài toán vô tỉ theo dạng trên ? Bài 1. Giải phương trình : ( ) ( ) ( )2 2 2 1 2 4 4 4 3 2 9 3 0x x x x x+ + + + + + + = Giải: ( ) ( )( ) ( ) ( )( ) ( ) ( ) 2 2 2 1 2 2 1 3 3 2 3 3 2 1 3x x x x f x f x⇔ + + + + = − + − + ⇔ + = − Xét hàm số ( ) ( )2 2 3f t t t= + + , là hàm đồng biến trên R, ta có 1 5 x = − Bài 2. Giải phương trình 3 2 23 4 5 6 7 9 4x x x x x− − + = + − Giải . Đặt 23 7 9 4y x x= + − , ta có hệ : ( ) ( ) 3 2 33 2 3 4 5 6 1 1 7 9 4 x x x y y y x x x x y  − − + = ⇒ + = + + + + − = Xét hàm số : ( ) 3 f t t t= + , là hàm đơn điệu tăng. Từ phương trình ( ) ( ) ( ) 23 5 1 1 1 7 9 4 1 5 2 x f y f x y x x x x x = =  +  ⇔ = + ⇔ + = + − ⇔ − ±   =  Bài 3. Giải phương trình : 33 6 1 8 4 1x x x+ = − − V. PHƯƠNG PHÁP LƯỢNG GIÁC HÓA 1. Một số kiến thức cơ bản:  Nếu 1x ≤ thì có một số t với ; 2 2 t π π− −  ∈   sao cho : sint x= và một số y với [ ]0;y π∈ sao cho cosx y=  Nếu 0 1x≤ ≤ thì có một số t với 0; 2 t π  ∈   sao cho : sint x= và một số y với 0; 2 y π  ∈    sao cho cosx y=  Với mỗi số thực x có ; 2 2 t π π  ∈ − ÷   sao cho : tanx t=  Nếu : x, y là hai số thực thỏa: 2 2 1x y+ = , thì có một số t với 0 2t π≤ ≤ , sao cho sin , cosx t y t= = Từ đó chúng ta có phương pháp giải toán :  Nếu : 1x ≤ thì đặt sint x= với ; 2 2 t π π− −  ∈   hoặc cosx y= với [ ]0;y π∈  Nếu 0 1x≤ ≤ thì đặt sint x= , với 0; 2 t π  ∈   hoặc cosx y= , với 0; 2 y π  ∈     Nếu : x, y là hai số thực thỏa: 2 2 1x y+ = , thì đặt sin , cosx t y t= = với 0 2t π≤ ≤ 20
  • 21.  Nếu x a≥ , ta có thể đặt : sin a x t = , với ; 2 2 t π π  ∈ − ÷   , tương tự cho trường hợp khác  x là số thực bất kỳ thi đặt : tan , ; 2 2 x t t π π  = ∈ − ÷   Tại sao lại phải đặt điều kiện cho t như vậy ? Chúng ta biết rằng khi đặt điều kiện ( )x f t= thì phải đảm bảo với mỗi x có duy nhất một t , và điều kiện trên để đảm bào điều này . (xem lại vòng tròn lượng giác ) 2. Xây dựng phương trình vô tỉ bằng phương pháp lượng giác như thế nào ? Từ công phương trình lượng giác đơn giản: cos3 sint t= , ta có thể tạo ra được phương trình vô tỉ Chú ý : 3 cos3 4cos 3cost t t= − ta có phương trình vô tỉ: 3 2 4 3 1x x x− = − (1) Nếu thay x bằng 1 x ta lại có phương trình : 2 2 2 4 3 1x x x− = − (2) Nếu thay x trong phương trình (1) bởi : (x-1) ta sẽ có phương trình vố tỉ khó: 3 2 2 4 12 9 1 2x x x x x− + − = − (3) Việc giải phương trình (2) và (3) không đơn giản chút nào ? Tương tự như vậy từ công thức sin 3x, sin 4x,…….hãy xây dựng những phương trình vô tỉ theo kiểu lượng giác . 3. Một số ví dụ Bài 1. Giải phương trình sau : ( ) ( ) 2 3 32 2 1 1 1 1 1 33 x x x x − + − + − − = +    Giải: Điều kiện : 1x ≤ Với [ 1;0]x∈ − : thì ( ) ( ) 3 3 1 1 0x x+ − − ≤ (ptvn) [0;1]x∈ ta đặt : cos , 0; 2 x t t π  = ∈   . Khi đó phương trình trở thành: 1 1 2 6 cos 1 sin 2 sin cos 2 6 x t t t   + = + ⇔ = ÷   vậy phương trình có nghiệm : 1 6 x = Bài 2. Giải các phương trình sau : 1) 1 2 1 2 1 2 1 2 1 2 1 2 x x x x x x − + − + + = + + − HD: 1 2cos tan 1 2cos x x x + = − 2) ( )2 2 1 1 1 2 1x x x+ − = + − Đs: 1 2 x = 3) 3 3 2x x x− = + HD: chứng minh 2x > vô nghiệm Bài 3 . Giải phương trình sau: 3 6 1 2x x+ = Giải: Lập phương 2 vế ta được: 3 3 1 8 6 1 4 3 2 x x x x− = ⇔ − = Xét : 1x ≤ , đặt [ ]cos , 0;x t t π= ∈ . Khi đó ta được 5 7 cos ;cos ;cos 9 9 9 S π π π  =     mà phương trình bậc 3 có tối đa 3 nghiệm vậy đó cũng chính là tập nghiệm của phương trình. Bài 4. .Giải phương trình 2 2 1 1 1 x x   + ÷ −  Giải: đk: 1x > , ta có thể đặt 1 , ; sin 2 2 x t t π π  = ∈ − ÷   21
  • 22. Khi đó ptt: ( )2 cos 0 1 1 cot 1 1 sin sin2 2 t t x t = + = ⇔  = −  Phương trình có nghiệm : ( )2 3 1x = − + Bài 5 .Giải phương trình : ( ) ( ) 222 2 2 11 1 2 2 1 xx x x x x ++ + = + − Giải: đk 0, 1x x≠ ≠ ± Ta có thể đặt : tan , ; 2 2 x t t π π  = ∈ − ÷   Khi đó pttt. ( )2 2sin cos2 cos2 1 0 sin 1 sin 2sin 0t t t t t t+ − = ⇔ − − = Kết hợp với điều kiện ta có nghiệm 1 3 x = Bài tập tổng hợp Giải các phương trình sau ( ) 33 2 2 1 2 2x x x x+ − = − 2 2 2 30 2007. 30 4 2007 30. 2007x x x− − + = 2 12 8 2 4 2 2 9 16 x x x x − + − − > + 33 3 1 1 2x x x− + + = 3 3 1 2 1x x x+ + = + 4 5 3 1 2 7 3x x x x+ + + = + + + ( )2 2 3 1 3 1x x x x+ + = + + 4 3 10 3 2x x− − = − (HSG Toàn Quốc 2002) ( ) ( ) ( ) ( )2 2 5 2 10x x x x x− − = + − − 23 4 1 2 3x x x+ = − + − 2 33 1 3 2 3 2x x x− + − = − 2 3 2 11 21 3 4 4 0x x x− + − − = (OLYMPIC 30/4-2007) 2 2 2 2 2 1 3 2 2 2 3 2x x x x x x x− + − − = + + + − + 2 2 2 16 18 1 2 4x x x x+ + + − = + 2 2 3 3 2 2 3 1 x x x x x + + + + = + 12 2 1 3 9x x x+ − = + 3 244 1 1x x x x+ + = + + 2 4 3 3 4 3 2 2 1x x x x x+ + = + + − 3 2 4 1 1 1 1x x x x x− + + + + = + − ( ) ( ) ( )2 2 4 2 4 16 2 4 16 2 9 16x x x x+ + − + − = + 2 (2004 )(1 1 )x x x= + − − ( 3 2)( 9 18) 168x x x x x+ + + + = 2 4 23 3 1 1 3 x x x x− + = − + + ( ) ( ) 2 2233 3 2 1 3 1 1 0x x x+ + − + − = 2 2008 4 3 2007 4 3x x x− + = − ( ) ( )2 2 3 2 1 1 1 3 8 2 1x x x x+ − = + + + 2 12 1 36x x x+ + + = ( ) 3 3 4 1 1 2 2 1x x x x− + = + + 1 1 1 2 1 3 x x x x x x − + = − + − 2 2 5 14 9 20 5 1x x x x x− + − − − = + 33 6 1 8 4 1x x x+ = − − ( ) ( )215 30 4 2004 30060 1 1 2 x x x− = + + 24 9 7 7 28 x x x + = + 2 2 4 4 10 8 6 10x x x x− − = − − 3 x x x x− = + CHUYÊN ĐỀ: PHƯƠNG TRÌNH VÔ TỶ 22
  • 23. I. PHƯƠNG PHÁP BIỂN ĐỔI TƯƠNG ĐƯƠNG Dạng 1 : Phương trình (*) 0 x D A B A B A B ∈ = ⇔ = ≥ ⇔  = Lưu ý: Điều kiện (*) được chọn tuỳ thuôc vào độ phức tạp của 0A ≥ hay 0B ≥ Dạng 2: Phương trình 2 0B A B A B ≥ = ⇔  = Dạng 3: Phương trình +) 0 0 2 A A B C B A B AB C  ≥  + = ⇔ ≥  + + = (chuyển về dạng 2) +) ( )3 3 3 33 3 3 .A B C A B A B A B C+ = ⇒ + + + = và ta sử dụng phép thế : 3 3 A B C+ = ta được phương trình : 3 3 . .A B A B C C+ + = Bài 1: Giải phương trình: a) 2 1 1x x− = − b) 2 3 0x x− + = c) 2 1 1x x+ + = e) 3 2 1 3x x− + − = f) 3 2 1x x+ − − = g) 9 5 2 4x x+ = − + h) 3 4 2 1 3x x x+ − + = + i) 2 2 ( 3) 10 12x x x x+ − = − − II.PHƯƠNG PHÁP ĐẶT ẨN PHỤ Phương pháp đặt ẩn phụ thông thường. -Nếu bài toán có chứa ( )f x và ( )f x khi đó đặt ( )t f x= (với điều kiện tối thiểu là 0t ≥ . đối với các phương trình có chứa tham số thì nhất thiết phải tìm điều kiện đúng cho ẩn phụ). -Nếu bài toán có chứa ( )f x , ( )g x và ( ). ( )f x g x k= (với k là hằng số) khi đó có thể đặt : ( )t f x= , khi đó ( ) k g x t = -Nếu bài toán có chứa ( ) ( ) ; ( ). ( )f x g x f x g x± và ( ) ( )f x g x k+ = khi đó có thể đặt: ( ) ( )t f x g x= ± suy ra 2 ( ). ( ) 2 t k f x g x − = -Nếu bài toán có chứa 2 2 a x− thì đặt sinx a t= với 2 2 t π π − ≤ ≤ hoặc cosx a t= với 0 t π≤ ≤ -Nếu bài toán có chứa 2 2 x a− thì đặt sin a x t = với { }; 0 2 2 t π π  ∈ −   hoặc cos a x t = với [ ]0; 2 t π π   ∈     Bài 2: Tìm m để phương trình sau có nghiệm: 2 2 3 2 2x x m x x− + − = + − Bài 3: Cho phương trình: 2 1x x m− − = -Giải phương trình khi m=1 -Tìm m để phương trình có nghiệm. Bài 4: Cho phương trình: 2 2 3x mx x m+ − = − -Giải phương trình khi m=3 -Với giá trị nào của m thì phương trình có nghiệm. 23
  • 24. -Nếu bài toán có chứa 2 2 x a+ ta có thể đặt .tanx a t= với ; 2 2 t π π  ∈ − ÷   24
  • 25. Bài 1: Giải phương trình: a) 2 2 2 8 12 2x x x x+ + + = − b) 2 2 2 5 2 3 9 3 3x x x x− + + = − − c) 2 2 4 6 2 8 12x x x x− + = − + d) 2 2 3 15 2 5 1 2x x x x+ + + + = e) 2 ( 4)( 1) 3 5 2 6x x x x+ + − + + = f) 2 2 2 5 2 2 2 5 6 1x x x x+ + − + − = g) 2 2 3 2 2 2 6 2 2x x x x+ + − + + = − h) 2 2 11 31x x+ + = i) 2 ( 5)(2 ) 3 3x x x x+ − = + Bài 2: Giải phương trình: a) ( ) ( ) 33 2 2 1 2 1x x x x+ − = − b) ( ) ( ) 3 32 2 1 1 1 1 2 1x x x x + − − − + = + −    c) 2 2 1 2 1 2 1 0x x x x− − − − + = d) 6 4 2 2 64 112 56 7 2 1x x x x− + − = − e) 2 35 121 x x x + = − f) ( ) ( ) ( ) 1 3 1 4 3 3 3 x x x x x + − + + − = − − Bài 4: Cho phương trình: 2 1 1 1 m x x + = − -Giải phương trình với 2 2 3 m = + -Tìm m để phương trình có nghiệm. Bài 5: Cho phương trình: ( )2 2 2 2 2 3 0x x x x m− + − − − = -Giải phương trình với m = 9 -Tìm m để phương trình có nghiệm. 2. Phương pháp đặt ẩn phụ không hoàn toàn Là việc sử dụng một ẩn phụ chuyển phương trình ban đầu thành một phương trình với một ẩn phụ nhưng các hệ số vẫn còn chứa x. -Từ những phương trình tích ( )( )1 1 1 2 0x x x+ − + − + = ,( )( )2 3 2 3 2 0x x x x+ − + − + = Khai triển và rút gọn ta sẽ được những phương trình vô tỉ không tầm thường chút nào, độ khó của phương trình dạng này phụ thuộc vào phương trình tích mà ta xuất phát. Từ đó chúng ta mới đi tìm cách giải phương trình dạng này .Phương pháp giải được thể hiện qua các ví dụ sau . Bài 1. Giải phương trình : ( )2 2 2 3 2 1 2 2x x x x+ − + = + + Giải: 2 2t x= + , ta có : ( )2 3 2 3 3 0 1 t t x t x t x = − + − + = ⇔  = − 25
  • 26. Bài 2. Giải phương trình : ( ) 2 2 1 2 3 1x x x x+ − + = + Giải: Đặt : 2 2 3, 2t x x t= − + ≥ Khi đó phương trình trở thnh : ( ) 2 1 1x t x+ = + ( )2 1 1 0x x t⇔ + − + = Bây giờ ta thêm bớt , để được phương trình bậc 2 theo t có ∆ chẵn ( ) ( ) ( ) ( )2 2 2 2 3 1 2 1 0 1 2 1 0 1 t x x x t x t x t x t x = − + − + + − = ⇔ − + + − = ⇔  = − Từ một phương trình đơn giản : ( )( )1 2 1 1 2 1 0x x x x− − + − − + + = , khai triển ra ta sẽ được pt sau Bài 3. Giải phương trình sau : 2 4 1 1 3 2 1 1x x x x+ − = + − + − Giải: Nhận xét : đặt 1t x= − , pttt: 4 1 3 2 1x x t t x+ = + + + (1) Ta rt 2 1x t= − thay vo thì được pt: ( ) ( )2 3 2 1 4 1 1 0t x t x− + + + + − = Nhưng không có sự may mắn để giải được phương trình theo t ( ) ( ) 2 2 1 48 1 1x x∆ = + + − + − không có dạng bình phương . Muốn đạt được mục đích trên thì ta phải tách 3x theo ( ) ( ) 2 2 1 , 1x x− + Cụ thể như sau : ( ) ( )3 1 2 1x x x= − − + + thay vào pt (1) ta được: Bài 4. Giải phương trình: 2 2 2 4 4 2 9 16x x x+ + − = + Giải . Bình phương 2 vế phương trình: ( ) ( ) ( )2 2 4 2 4 16 2 4 16 2 9 16x x x x+ + − + − = + Ta đặt : ( )2 2 4 0t x= − ≥ . Ta được: 2 9 16 32 8 0x t x− − + = Ta phải tách ( ) ( )2 2 2 9 2 4 9 2 8x x xα α α= − + + − làm sao cho t∆ có dạng chình phương . Nhận xét : Thông thường ta chỉ cần nhóm sao cho hết hệ số tự do thì sẽ đạt được mục đích. Bài tập: Giải các phương trình sau: a) 3 3 (4 1) 1 2 2 1x x x x− + = + + b) 2 2 1 2 2x x x x− = − c) 2 2 1 2 2x x x x− = + d) 2 2 4 ( 2) 2 4x x x x x+ = + − + 3. Phương pháp đặt ẩn phụ chuyển về hệ. a) Dạng thông thường: Đặt ( ) ( ),u x v xα β= = và tìm mối quan hệ giữa ( )xα và ( )xβ từ đó tìm được hệ theo u,v. Chẳng hạn đối với phương trình: ( ) ( )m ma f x b f x c− + + = ta có thể đặt: ( ) ( ) m m u a f x v b f x  = −  = + từ đó suy ra m m u v a b+ = + . Khi đó ta có hệ m m u v a b u v c  + = +  + = Bài tập: Giải các phương trình sau: a) 3 2 1 1x x− = − − b) 3 9 2 1x x− = − − c) 2 1 ( 1) 0x x x x x x− − − − + − = b) Dạng phương trình chứa căn bậc hai và lũy thừa bậc hai: 26
  • 27. 2 ( )ax b c dx e xα β+ = + + + với d ac e bc α β = +  = + Cách giải: Đặt: dy e ax b+ = + khi đó phương trình được chuyển thành hệ: ( ) ( ) 2 22 ( ) dy e ax bdy e ax b dy e c dx e x c dy e x dy eα β α β  + = + + = +  ⇔  + = + + + + = − + + −  ->giải Nhận xét: Dể sử dụng được phương pháp trên cần phải khéo léo biến đổi phương trình ban đầu về dạng thỏa mãn điều kiện trên để đặt ẩn phụ.Việc chọn ;α β thông thường chúng ta chỉ cần viết dưới dạng : ( ) ' ' n n x p a x bα β γ+ = + + là chọn được. c) Dạng phương trình chứa căn bậc ba và lũy thừa bậc ba. ( ) 33 ax b c dx e xα β+ = + + + với d ac e bc α β = +  = + Cách giải: Đặt 3 dy e ax b+ = + khi đó phương trình được chuyển thành hệ: ( ) ( ) ( ) ( ) 3 33 3 3 3 ( ) ( ) dy e ax bdy e ax b c dy e acx bc dy e c dx e x c dx e ac d x dy bcc dx e x dy e  + = ++ = + + = +   ⇔ ⇔   + = + + + + = − + ++ = − + + −    α β α β Bài tập: Giải các phương trình sau: 1) 2 1 4 5x x x+ = + + 2) 2 3 1 4 13 5x x x+ = − + − 3) 3 3 2 3 3 2x x+ = − 4) 24 9 7 7 0 28 x x x x + = + > 5) 3 3 1 2 2 1x x+ = − 6) ( )3 33 3 35 35 30x x x x− + − = 7) 2 4 13 5 3 1 0x x x− + + + = 8) 2 4 13 5 3 1 0x x x− + + + = ( ) ( )215 30 4 2004 30060 1 1 2 x x x− = + + 3 23 3 5 8 36 53 25x x x− = − + − 9) 3 23 4 81 8 2 2 3 x x x x− = − + − 10) 33 6 1 8 4 1x x x+ = − − II. PHƯƠNG PHÁP HÀM SỐ Sử dụng các tính chất của hàm số để giải phương trình là dạng toán khá quen thuộc. Ta có 3 hướng áp dụng sau đây: Hướng 1: Thực hiện theo các bước: Bước 1: Chuyển phương trình về dạng: ( )f x k= Bước 2: Xét hàm số ( )y f x= Bước 3: Nhận xét: • Với 0 0( ) ( )x x f x f x k= ⇔ = = do đó 0x là nghiệm • Với 0 0( ) ( )x x f x f x k> ⇔ > = do đó phương trình vô nghiệm • Với 0 0( ) ( )x x f x f x k< ⇔ < = do đó phương trình vô nghiệm • Vậy 0x là nghiệm duy nhất của phương trình Hướng 2: thực hiện theo các bước Bước 1: Chuyển phương trình về dạng: ( ) ( )f x g x= Bước 2: Dùng lập luận khẳng định rằng ( )f x và g(x) có những tính chất trái ngược nhau và xác định 0x sao cho 0 0( ) ( )f x g x= 27
  • 28. Bước 3: Vậy 0x là nghiệm duy nhất của phương trình. Hướng 3: Thực hiện theo các bước: Bước 1: Chuyển phương trình về dạng ( ) ( )f u f v= Bước 2: Xét hàm số ( )y f x= , dùng lập luận khẳng định hàm số đơn điệu Bước 3: Khi đó ( ) ( )f u f v u v= ⇔ = Ví dụ: Giải phương trình : ( ) ( ) ( )2 2 2 1 2 4 4 4 3 2 9 3 0x x x x x+ + + + + + + = pt ( ) ( )( ) ( ) ( )( ) ( ) ( ) 2 2 2 1 2 2 1 3 3 2 3 3 2 1 3x x x x f x f x⇔ + + + + = − + − + ⇔ + = − Xét hàm số ( ) ( )2 2 3f t t t= + + , là hàm đồng biến trên R, ta có 1 5 x = − Bài tập: Giải phương trình: 2 4 1 4 1 1x x− + − = , 3 1 4 5x x x− = − − + , 2 1 3x x x− = + − , 2 3 1 2 2x x x x= − + − , 1 2 3x x− + + = , 2 2 1 3 4x x x− + + = − 28
  • 29. BAØI TAÄP : Baøi 1: Bình phöông hai veá : a) x 2 + 1 1x + = Hd: pt 4 2 0 1 1 1 2 0 1 5 2 x x x x x x x   = − ≤ ≤ ⇔ =− − − =  ± =  b)pt: 5 1 3 2 1 0 : 1 x x x dk x − − − − − = ≥ - Chuyeån veá ,bình phöông hai veá : x =2 ; x = 2/11( loaïi ) . Vaäy x=2 . c) : 9 5 2 4 : 2 pt x x dk x + = − + ≥ Bình phöông hai laà ta coù :ÑS x = 0 . d) : 16 9 7 : 0; 7 pt x x Ds x − + + = = − e) 2 2 :(4 1) 9 2 2 1 : 1/ 4 pt x x x x dk x − + = + + ≥ Bphöông hai lanà ta coù :ÑS x = 4/3 Baøi 2 : Daët Aån soá phuï : a) 2 2 3 3 3 6 3x x x x− + + − + = - Ñaët : - T=x 2 -3x+3 3 / 4 : 3 3 1 1; 2 pt t t t x ≥ + + = <=> = => = b) 22 1 1 0 3 : 0 1 x x x x dk x + − = + − = ≤ ≤ - Ñaët : 2 2 1 1 ; 0 2 t t x x t x x − = + − ≥ => − = ptt 2 -3t +2 =0 t =1 ; t=2 Vn t=1  x=0 ; x=1 . c) 2 2 3 1 3 2 2 5 3 16x x x x x+ + + = + + + − HDÑS: 29
  • 30. ÑK : 2 2 1 2 3 1 0 3 4 2 2 5 3 5 3. x t x x t x x x pt t x ≥ − = + + + ≥ => = + + + + <=> = <=> = 2 2 2 2 ) 7 2 3 3 19 . 2 7 / 4 5 3 13 4 1; 2 d x x x x x x t x x pt t t t t x x + + + + + = + + = + + ≥ <=> + + = + <=> = => = = − 3: ) 1 3 ( 1)(3 )Bai a x x x x m+ + − − + − = • Giaûi pt khi m=2 .** Tìm m pt coù nghieäm . • HDÑS : ÑK: 2 . 1 3 ; 2 2 2 : 2( ) 0( ) ) 2: 2 0 1, 3 2 t x x t vi a b a b a b t l a m t t x x t = + + − => ≤ ≤ + ≤ + ≤ + = = − = <=> => = − = = b) f(t) = -t 2 /2 + t +2 = m (1) . Laäp baûng bieán thieân : Tacoù : 2 2 2 2.m− ≤ ≤ 2 4: ) 9 9bai a x x x x m+ − = − + + Bình phöông : Ñaët t= (9 ) 0 9/ 2x x t− => ≤ ≤ KsHS 2 ( ) 2 9 ; 9/ 2 9/ 4 10f t t t o t Ds m= − + + ≤ ≤ − ≤ ≤ d) 4 44 4 4 6x x m x x m+ + + + + = HDÑS:Ñaët : 4 24 44 4 4 0 : 6 0 3 2 4 2 4 16 t x x m pt t t t l t x x m m x x = + + ≥ + − = =− <=> = => + + = <=> =− − + Laäp BBT : m>19VN; m=19: 1 ngh ;m<19pt2ngh. Baøi3: 1- BAØI TAÄP : I- GIAÛI PT: 3 3 1) 2 2 3 1X X− + − = Laäp phöông hai veá ta coù : x 3 -4x 2 +5x-2 =0 x=1 ; x=2 . Thöû laïi x=1 khoâng thoaû .Vaäy x=2 . 30
  • 31. 3 3 2) 34 3 1X X+ − − = Laäp phöông hai veá ta coù : x 2 +31x-1830 =0 x=-1061 ; x=75 . 3 3 2) 2 2 3 1X X+ + − = -Laäp phöông hai veá ta coù : x 3 -4x 2 +5x-2 =0 x=1 ; x=2 . Thöû laïi x=1 khoâng thoaû .Vaäy x=2 . 3 3 3 3) 1 2 2 3X X x− + − = − -Laäp phöông hai veá ta coù : pt x=1 ; x=2 ;x=3/2. Thöû laïi Ñeàu thoaû . 3 3 3 4) 2 2 2 9X X x+ + − = --Laäp phöông hai veá ta coù : pt x=0 ; x=3 ;x=-6/5. Thöû laïi Ñeàu thoaû . 32 2 2 23 3 5) ( ) 2 ( ) 0X a X a x a a+ + + − = − ≠ pt 18X = 14a  x=7a/ 9 ; a# 0 . Thöû laïi thoaû. II- PHÖÔNG TRÌNH CAÊN THÖÙC CHÖÙA THAM SOÁ m : Baøi 1: 2 1x x m+ = + : BLs ngh pt. HdÑS : [ )1; : ( ) 2 1 D xet f x x x = − +∞ − = + − Tính ñaïo haøm : Baûng bieán thieân ,Ta coù : m<1 : 1ngh; m=1: coù 2ngh: 1<m<2: 2ngh m=2 : 2ngh ; 2<m< +∞ : 0 Vn . BAÁT PHÖÔNG TRÌNH CAÊN THÖÙC KIEÁN THÖÙC CAÀN NHÔÙ : • Daïng cô baûn : 2 2 2 2 0 0 0 0 0 0 0 0 0 0 A A B B A B A A B B A B A B A B B A B A B A B B A B  ≥  < ⇔ >  <  ≥  ≤ ⇔ ≥  ≤  ≥  <> ⇔ ≥  >  ≥  ≤≥ ⇔ >  ≥ • Daïng khaùc : - Coù nhieàu caên thöùc :Ñaët ÑK – Luyõ thöøa- khöû caên – Döa veå bpt cô baûn nhö caùc daïng treân . Chuù yù : - Hai veá khoâng aâm ta ñ7ôïc bình phöông – Hai veá laø soá thöïc ta ñöïôc laäp phöông . 31
  • 32. BAØI TAÄP : GIAÛI CAÙC BAÁT PHÖÔNG TRÌNH 1-Pt : 2 3 2x x+ ≥ − pt -3/2 3 2 2x≤ ≤ + 32
  • 33. PHÖÔNG TRÌNH CAÊN THÖÙC III. PHƯƠNG PHÁP BIỂN ĐỔI TƯƠNG ĐƯƠNG Dạng 1 : Phương trình (*) 0 x D A B A B A B ∈ = ⇔ = ≥ ⇔  = Lưu ý: Điều kiện (*) được chọn tuỳ thuôc vào độ phức tạp của 0A ≥ hay 0B ≥ Dạng 2: Phương trình 2 0B A B A B ≥ = ⇔  = Dạng 3: Phương trình +) 0 0 2 A A B C B A B AB C  ≥  + = ⇔ ≥  + + = (chuyển về dạng 2) +) ( )3 3 3 33 3 3 .A B C A B A B A B C+ = ⇒ + + + = và ta sử dụng phép thế : 3 3 A B C+ = ta được phương trình : 3 3 . .A B A B C C+ + = Bài tập trong các đề thi tuyển sinh. Bài 1 : a)(ĐHXD) Giải pt 2 6 6 2 1x x x− + = − b) (CĐSP MG 2004) 2 4 3 2 5x x x− + − = − c) (CĐSP NINH BÌNH) 3 2 7 1x x− − + = d) (CĐ hoá chất) 8 3x x x+ − = + e) (CĐ TP 2004) 2 2 1 7x x− − = g) (CĐSP bến tre) 5 1 3 2 1 0x x x− − − − − = h) (CĐ truyền hình 2007) 2 2 7 5 3 2x x x x x− + + = − − ĐS: a) x=1. b) x=14/5 c) x=9. d)x=1 e) x=5 g) x=2 h) x=-1. Bài 2: a)(ĐHNN-2001) Giải phương trình 1 4 ( 1)(4 ) 5.x x x x+ + − + + − = b) (CĐ Nha trang 2002) : 2 5 ( 2)(5 ) 4x x x x+ + − + + − = Hdẫn: a) ĐK: -1≤x≤4. Đặt t= 1 4 0x x+ + − > . Giải được t=-5 (loại), t=3. Giải t=3 được x=0. b) x= 3 3 5 2 ± Bài 3 a)(ĐHQG KD-2001) Giải phương trình 2 4 1 4 1 1x x− + − = . b) (CĐXD 2003) 3 3 3 2 1 2 2 2 3 0x x x+ + + + + = 33
  • 34. Hdẫn: a) ĐK: x≥1/2 Xét hàm số y= 2 4 1 4 1x x− + − . HSĐB trên [1/2;+∞). Và f(1/2)=1. Vậy phương trình có nghiệm duy nhất x=1/2. b)x=-1 là nghiệm . Các hàm số y= 3 2 1x + ; y= 3 2 2x + ; y= 3 2 3x + ĐB Bài 4 : Giải pt 2 2 2 8 6 1 2 2x x x x+ + + − = + . ĐK : x ≤-3,x=-1,x≥1. -Với x=-1 Thoả mãn pt -Với x≤-3 thì VP<0 loại -Với x≥1 pt 2 ( 1)(2 6) ( 1)( 1) 2 ( 1) 2 6 1 2 1 x x x x x x x x ⇔ + + + + − = + ⇔ + + − = + Tiếp tục bình phương 2 vế thu được x=1. Vậy pt có 2 nghiệm x=1 ; x=-1. Bài 5 : (ĐH mỏ điạ chất) Giải pt 2 2 4 2 3 4x x x x+ − = + − ĐK : 2x ≤ . Đặt t= 2 4x x+ − . Giải được t=2 ; t=-4/3. +t=2 được x=0, x=2 +t=-4/3 được 2 14 2 14 ; 3 3 x x − − − + = = (loại) KL : Pt có 3 nghiệm. Bài 6 : (HV CNBCVT) Giải pt 3 4 1 3 2 5 x x x + + − − = . Giải : ĐK : x≥2/3. Trục căn thức ta được 3 3 ( 4 1 3 2) 4 1 3 2 5 5 x x x x x x + + = + + − ⇔ + + − = . PT trên có nghiệm x=2. HS y= 4 1 3 2x x+ + − ĐB do vậy x=2 là nghiệm duy nhất. Bài 7: Giải phương trình 3(2 2) 2 6x x x+ − = + + . ĐK: x≥2. pt 2(3 ) 6 2 2 2(3 )( 6 2 2) 8(3 ) 3 6 2 2 4 x x x x x x x x x x ⇔ − = + − − ⇔ − + + − = − = ⇔  + + − = KL: x=3; x= 11 15 2 − 34
  • 35. Bài 8: Giải phương trình 2 7 7x x+ + = ĐK:x≥-7. Đặt 2 7 0 7t x t x= + ≥ ⇒ = + . Phương trình trở thành 2 2 2 2 ( ) ( )( 1) 0 7 x t x t x t x t x t t x  + = ⇒ − = − + ⇔ + − + = = + Giải được x=2; x= 1 29 2 − a) + = −3 3 1 2 2 1x x + = − = − ⇔ + = 3 3 33 1 2 2 1 2 1 1 2 x x y x y x - Ph¬ng tr×nh ®îc chuyÓn thµnh hÖ   =  = =   + = + = + =  − +   ⇔ ⇔ ⇔ = =   + = − = − − + + + =      − − + = = = 33 3 3 3 3 2 2 3 1 1 21 2 1 2 1 5 21 2 2( ) 2 0( ) 1 51 2 2 x y x y x yx y x y x y y x x y x y x xy y vn x y x y - VËy ph¬ng tr×nh ®· cho cã 3 nghiÖm. c) 2 23 3 3(2 ) (7 ) (7 )(2 ) 3x x x x− + + − − − = -Ñaët : 2 23 32 . 3 33 7 9 3 1; 2 1; 6 2 u v uvu x pt v x u v u v u v x uv            + − == − <=> = + + = + = <=> <=> = = => = − = d) 3 2 1 1x x− = − − .ÑK : x 1≥ 3 2 1; 0 1 0;1; 2; 1;0;33 2 1 1;2;10 u x v x v u v u v u v x        = − = − ≥ = − => <=> = − = + = = Bài 9: Giải phương trình 2 2 2 2 4 2 2x x x x− − + = − − + ĐK: x≥2. Đặt 2 2 2 2 2 4 2t x x t x x= − − + ⇒ = − − + . Thế vào phương trình giải được t=1; t=-2. từ đó giải được x=2. Bài 10: (Tham khảo 2002) giải phương trình 2 4 4 2 12 2 16x x x x+ + − = − + − 35
  • 36. ĐK:x≥4. Phương trình 2 4 4 ( 4 4) 12x x x x⇔ + + − = + + − − Đặt t= 4 4x x+ + − ≥0. giải phương trình ẩn t được t=4; t=-3 (loại). Giải được x=5. Bài 11 : a)(CĐSP 2004) Giải pt 3 2 1 2 1 2 x x x x x + + − + − − = b) (ĐH-KD-2005) 2 2 2 1 1 4x x x+ + + − + = a) ĐK ; x≥1. Pt 3 1 1 1 1 2 x x x + ⇔ − + + − − = . Xét 1≤x≤2 : giải được nghiệm x=1 xét x>2 giải được x=5 b)x=3 3. Phương pháp đặt ẩn phụ không hoàn toàn  Từ những phương trình tích ( )( )1 1 1 2 0x x x+ − + − + = ,( )( )2 3 2 3 2 0x x x x+ − + − + = Khai triển và rút gọn ta sẽ được những phương trình vô tỉ không tầm thường chút nào, độ khó của phương trình dạng này phụ thuộc vào phương trình tích mà ta xuất phát . Từ đó chúng ta mới đi tìm cách giải phương trình dạng này .Phương pháp giải được thể hiện qua các ví dụ sau . Bài 1. Giải phương trình : ( )2 2 2 3 2 1 2 2x x x x+ − + = + + Giải: 2 2t x= + , ta có : ( )2 3 2 3 3 0 1 t t x t x t x = − + − + = ⇔  = − Bài 2. Giải phương trình : ( ) 2 2 1 2 3 1x x x x+ − + = + Giải: Đặt : 2 2 3, 2t x x t= − + ≥ Khi đó phương trình trở thnh : ( ) 2 1 1x t x+ = + ( )2 1 1 0x x t⇔ + − + = Bây giờ ta thêm bớt , để được phương trình bậc 2 theo t có ∆ chẵn : ( ) ( ) ( ) ( )2 2 2 2 3 1 2 1 0 1 2 1 0 1 t x x x t x t x t x t x = − + − + + − = ⇔ − + + − = ⇔  = − 36